Está en la página 1de 87

TEORA DE NMEROS

D. Francisco Medrano
con series de ejercicios corregidos

ndice
1. Divisibilidad
1.1. Notaciones . . . . . . . . . . . . . . . .
1.2. Algoritmo de la divisin . . . . . . . .
1.3. Mximo comn divisor (mcd) . . . . .
1.4. El Algoritmo de Euclides . . . . . . . .
1.5. Teorema fundamental de la Aritmtica
1.6. Serie de ejercicios 1 . . . . . . . . . . .
2. Funciones aritmticas
2.1. Funciones multiplicativas
2.2. La funcin de Euler . . .
2.3. La funcin de Mbius . . .
2.4. Producto de convolucin .
2.5. Las funciones y . . . .
2.6. Nmeros perfectos . . . .
2.7. Funcin zeta de Riemann .
2.8. Serie de ejercicios 2 . . . .

.
.
.
.
.
.
.
.

.
.
.
.
.
.
.
.

3. Congruencias
3.1. Teorema chino del resto . . . .
3.2. Teoremas de Fermat y Euler .
3.3. Teorema de Wilson . . . . . .
3.4. Teorema de Lagrange . . . . .
3.5. Races Primitivas . . . . . . .
3.6. Indices (Logaritmos discretos)
3.7. Esquema de Diffie-Helman . .
3.8. Serie de ejercicios 3 . . . . . .

.
.
.
.
.
.
.
.
.
.
.
.
.
.
.
.

4. Residuos cuadrticos
4.1. Smbolo de Legendre . . . . . .
4.2. Criterio de Euler . . . . . . . . .
4.3. Lema de Gauss . . . . . . . . . .
4.4. Ley de reciprocidad cuadrtica .
4.5. Serie de ejercicios 4 . . . . . . .
4.6. Smbolo de Jacobi . . . . . . . .
4.7. Serie de ejercicios 5 . . . . . . .

.
.
.
.
.
.
.
.
.
.
.
.
.
.
.
.
.
.
.
.
.
.
.

.
.
.
.
.
.
.
.
.
.
.
.
.
.
.
.
.
.
.
.
.
.
.

.
.
.
.
.
.
.
.
.
.
.
.
.
.
.
.
.
.
.
.
.
.
.

.
.
.
.
.
.
.
.
.
.
.
.
.
.
.
.
.
.
.
.
.
.
.

.
.
.
.
.
.
.
.
.
.
.
.
.
.
.
.
.
.
.
.
.
.
.
.
.
.
.
.
.

.
.
.
.
.
.
.
.
.
.
.
.
.
.
.
.
.
.
.
.
.
.
.
.
.
.
.
.
.

.
.
.
.
.
.
.
.
.
.
.
.
.
.
.
.
.
.
.
.
.
.
.
.
.
.
.
.
.

.
.
.
.
.
.
.
.
.
.
.
.
.
.
.
.
.
.
.
.
.
.
.
.
.
.
.
.
.

.
.
.
.
.
.
.
.
.
.
.
.
.
.
.
.
.
.
.
.
.
.
.
.
.
.
.
.
.

.
.
.
.
.
.
.
.
.
.
.
.
.
.
.
.
.
.
.
.
.
.
.
.
.
.
.
.
.

.
.
.
.
.
.
.
.
.
.
.
.
.
.
.
.
.
.
.
.
.
.
.
.
.
.
.
.
.

.
.
.
.
.
.
.
.
.
.
.
.
.
.
.
.
.
.
.
.
.
.
.
.
.
.
.
.
.

.
.
.
.
.
.
.
.
.
.
.
.
.
.
.
.
.
.
.
.
.
.
.
.
.
.
.
.
.

.
.
.
.
.
.
.
.
.
.
.
.
.
.
.
.
.
.
.
.
.
.
.
.
.
.
.
.
.

.
.
.
.
.
.
.
.
.
.
.
.
.
.
.
.
.
.
.
.
.
.
.
.
.
.
.
.
.

.
.
.
.
.
.

5
5
5
5
5
6
7

.
.
.
.
.
.
.
.

8
8
9
10
11
13
14
15
16

.
.
.
.
.
.
.
.

18
18
19
20
20
22
24
25
25

.
.
.
.
.
.
.

26
26
27
28
29
31
31
33

5. Formas cuadrticas
5.1. Suma de cuadrados . . . . .
5.2. Serie de ejercicios 6 . . . . .
5.3. Sumas de cuatro cuadrados .
5.4. Serie de ejercicios 7 . . . . .

.
.
.
.

.
.
.
.

.
.
.
.

.
.
.
.

.
.
.
.

.
.
.
.

.
.
.
.

.
.
.
.

.
.
.
.

.
.
.
.

.
.
.
.

.
.
.
.

.
.
.
.

.
.
.
.

.
.
.
.

.
.
.
.

.
.
.
.

.
.
.
.

35
36
38
38
40

6. Aproximaciones diofnticas
6.1. Teorema de Dirichlet . .
6.2. Fracciones continuas . .
6.3. Serie de ejercicios 8 . . .
6.4. Teorema de Liouville . .
6.5. Nmeros trascendentales
6.6. Serie de ejercicios 9 . . .

.
.
.
.
.
.

.
.
.
.
.
.

.
.
.
.
.
.

.
.
.
.
.
.

.
.
.
.
.
.

.
.
.
.
.
.

.
.
.
.
.
.

.
.
.
.
.
.

.
.
.
.
.
.

.
.
.
.
.
.

.
.
.
.
.
.

.
.
.
.
.
.

.
.
.
.
.
.

.
.
.
.
.
.

.
.
.
.
.
.

.
.
.
.
.
.

.
.
.
.
.
.

.
.
.
.
.
.

42
42
44
47
47
48
51

.
.
.
.
.
.
.

56
56
58
59
61
61
64
66

7. Cuerpos cuadrticos
7.1. Cuerpo de nmeros . . .
7.2. Cuerpos cuadrticos . . .
7.3. Unidades . . . . . . . . .
7.4. Serie de ejercicios 10 . .
7.5. El cuerpo Gaussiano Q(i)
7.6. Ecuaciones de Pell . . . .
7.7. Serie de ejercicios 11 . .

.
.
.
.
.
.
.
.
.
.
.
.
.

.
.
.
.
.
.
.
.
.
.
.
.
.

.
.
.
.
.
.
.

.
.
.
.
.
.
.

.
.
.
.
.
.
.

.
.
.
.
.
.
.

.
.
.
.
.
.
.

.
.
.
.
.
.
.

.
.
.
.
.
.
.

.
.
.
.
.
.
.

.
.
.
.
.
.
.

.
.
.
.
.
.
.

.
.
.
.
.
.
.

.
.
.
.
.
.
.

.
.
.
.
.
.
.

.
.
.
.
.
.
.

.
.
.
.
.
.
.

.
.
.
.
.
.
.

.
.
.
.
.
.
.

8. La conjetura ABC
8.1. ABC para los polinomios . . . . . . . . . . . . . . . . . . . .
8.2. Preparativos para la prueba del teorema ABC para los polinomios . . . . . . . . . . . . . . . . . . . . . . . . . . . . . .
8.3. ABC para los enteros? . . . . . . . . . . . . . . . . . . . . .

67
67

9. Soluciones e indicaciones de las series de ejercicios


9.1. Serie de ejercicios 1 . . . . . . . . . . . . . . . .
9.2. Serie de ejercicios 2 . . . . . . . . . . . . . . . .
9.3. Serie de ejercicios 3 . . . . . . . . . . . . . . . .
9.4. Serie de ejercicios 4 . . . . . . . . . . . . . . . .
9.5. Serie de ejercicios 5 . . . . . . . . . . . . . . . .
9.6. Serie de ejercicios 6 . . . . . . . . . . . . . . . .
9.7. Serie de ejercicios 7 . . . . . . . . . . . . . . . .
9.8. Serie de ejercicios 8 . . . . . . . . . . . . . . . .
9.9. Serie de ejercicios 9 . . . . . . . . . . . . . . . .
9.10. Serie de ejercicios 10 . . . . . . . . . . . . . . .
9.11. Serie de ejercicios 11 . . . . . . . . . . . . . . .

73
73
74
76
77
78
79
81
82
83
85
86

.
.
.
.
.
.
.
.
.
.
.

.
.
.
.
.
.
.
.
.
.
.

.
.
.
.
.
.
.
.
.
.
.

.
.
.
.
.
.
.
.
.
.
.

.
.
.
.
.
.
.
.
.
.
.

.
.
.
.
.
.
.
.
.
.
.

.
.
.
.
.
.
.
.
.
.
.

69
70

Contenido

1.
1.1.

Divisibilidad
Notaciones

Con el fin de fijar las ideas: N = {1, 2, 3, ...}, Z = {..., 1, 0, 1, 2, ...} y decimos
que a|b (lase a divide b) si por definicin existe c N tal que b = ac.
Observacin 1. Una consecuencia directa de la definicin de divisibilidad
es que si c|a y a + b = d, entonces a|b c|d.

1.2.

Algoritmo de la divisin

Proposicin 1. Para cada a, b Z, b > 0, q, r Z tales que a = bq + r con


0 6 r < b.
Demostracin. Podemos suponer que a > 0. Pongamos q := max{s Z : bs
a} y r := a bq. Se verifica fcilmente, por definicin de q y r, el enunciado
de la proposicin.

1.3.

Mximo comn divisor (mcd)

Definicin 1. Decimos que d es un divisor comn de a y b si d|a y d|b. Llamamos mximo comn divisor de a y b, que notamos por mcd(a, b) o simplemente por (a, b), al ms grande de tales divisores comunes.
Proposicin 2. Sea d = mcd(a, b), entonces d es el entero positivo ms pequeo
bajo a forma ax + by con x, y Z.
Demostracin. Sea D = ax + by > 0, vemos que d|D y entonces d D. Supongamos ahora que D es minimal (bajo esa forma), por la proposicin 1
escribamos a = Dq + r, entonces r = a Dq = a(1 xq) + b(yq). Pero r < D,
como D es minimal entonces r = 0, lo que implica que D|a. De la misma
manera se muestra que D|b, de esta forma hemos mostrado que D es un
divisor comn de a y b, por definicin D d y entonces D = d.

1.4.

El Algoritmo de Euclides

Dados a y b enteros, el algoritmo siguiente nos permite encontrar el mcd(a, b)


por divisiones euclidianas sucesivas.
Se calculan sucesivamente los cocientes y restos de las siguientes divisiones:
5

a = bq1 + r1
b = r1 q2 + r2
..
.
rk2 = rk1 qk + rk
rk1 = rk qk+1 + rk+1

0 r1 < b,
0 r2 < r 1 ,
0 rk < rk1 ,
0 rk1 < rk .

Observemos que r1 > r2 > ... 0 implica que este algoritmo se detiene en
un nmero finito de etapas. Entonces, existe un k tal que rk+1 = 0.
Afirmacin: mcd(a, b) = rk .
En efecto, viendo las igualdades precedentes de abajo hacia arriba se observa fcilmente que rk |a y rk |b. De un modo similar, viendo las mismas
igualdades esta vez de arriba hacia abajo, vemos que cada divisor comn
de a y b divide tambin rk , lo que permite concluir que mcd(a, b) = (a, b) =
rk .

1.5.

Teorema fundamental de la Aritmtica

Definicin 2. p N es llamado nmero primo o simplemente primo si p , 1


y sus nicos divisores son 1 y p.
Lema 1. Si p es primo y p | ab, entonces p | a o p | b.
Demostracin. Supongamos que p - a, entonces mcd(p, a) = 1 y luego existen x, y Z tales que px + ay = 1. Multiplicando por b de ambos lados de la
igualdad: bpx + aby = b, luego como p | ab tenemos que p | b.
Teorema 1. (Teorema fundamental de la Aritmtica). Podemos escribir n
j
j
N, n > 1 como producto de nmeros primos n = p11 pkk .
Demostracin. Existencia: Sea n > 1, tomemos p1 el divisor de n distinto
de 1 ms pequeo que n (que puede repetirse), p1 es entonces primo. Pongamos n0 = pn1 y luego repitamos. La descomposicin en factores primos
queda entonces demostrada.
j

Unicidad: Supongamos dos descomposiciones de n = p11 pkk = q11 qll


i

en factores primos. Como pi | q11 qll , por el lema 1, pi | qm para un cierto


m {1, , l}, luego como pi y qm son primos se tiene que pi = qm . Luego dividimos n por pi = qm y repetimos este proceso, lo que demuestra la
unicidad.
6

Observacin 2. Para encontrar el mcd de a y b, consideramos sus descomposiciones en factores primos (aadiendo los factores primos de a a b
n
n
y viceversa por medio de potencias de cero si es necesario) a = p1 1 pk k ,
m
m
b = p1 1 pk k , 0 ni , mi i = 1, , k y luego la frmula es:
mcd(a, b) =

k
Y

mn(ni ,mi )

pi

i=1

Teorema 2. Existe una infinidad de nmeros primos.


Demostracin. Supongamos que p1 , , pn son los n primeros nmeros primos. Pongamos x = p1 pn + 1. Vemos que pk - x, k = 1, , n, de otra forma existira pi con i {1, 2, ..., n} tal que pi | x = p1 ...pn + 1 y por lo tanto
pi | (p1 ...pn + 1) p1 ...pn = 1 lo que es una contradiccin . Ahora bien, el
teorema fundamental de la Aritmtica nos dice que existe un primo q que
divide x, este primo es estrictamente ms grande que cualquier pi de la
lista {p1 , , pn }. Luego siempre podremos encontrar un primo ms grande
que uno dado, lo que muestra la infinitud de los nmeros primos.

1.6.

Serie de ejercicios 1

1. Mostrar que existe una infinidad de primos de la forma 4k + 3.


n

2. Un nmero primo de la forma 22 + 1 se llama nmero primo de Fermat. Mostrar que si 2n + 1 es primo, entonces es un primo de Fermat.
m

3. (i) Mostrar que si m > n 0, entonces mcd(22 + 1, 22 + 1) = 1. Indim


m
cacin: Escribir 22 + 1 = 22 1 + 2.
Sea pn el n-simo nmero primo y (x) el nmero de primos comprendidos entre 2 y un nmero x dado (que no es necesariamente
entero), es decir (x) es una funcin que indica cmo se encuentran
repartidos los nmeros primos.
n

(ii) Deducir de (i) que pn 22 + 1 y que (x) log log x para x 2.


4. Sean a, b, c N2 . Mostrar que:
mcd(mcm(a, b), mcm(a, c), mcm(b, c)) = mcm(mcd(a, b), mcd(a, c), mcd(b, c))
5. Mostrar que el n-simo nmero armnico Hn :=
para n 2

Pn

1
k=1 k

no es entero

2.

Funciones aritmticas

Definicin 3. Una funcin aritmtica es una funcin de N a valores en R o


bien en C.
Recordemos que llamamos parte entera de x R, que notamos por [x], al
nico entero que verifica x 1 < [x] x. La cantidad x [x], se llama parte
fraccionaria de x y se escribe {x}.
j
Lema
h 2.i Sea p primo, n N. Escribamos l = max{j 0 : p | n!}. Entonces
n
.
l p1

h i
Demostracin. Observemos que entre los nmeros 1, , n hay np que son
divisibles por
 p (considerar la lista de los mltiplos de p: p,2p,...,kp con
kp n),

n
p2

divisibles por p2 , etc. No es difcil de demostrar que x, y

R, [x + y] [x] + [y]. Entonces


l=

X
X
m=1 j=1, pj |m

1=

n
X

1=

j=1 m=1, pj |m

" #
X
n
j=1

pj

"

X
n
n

.

=

p1
pj
j=1

La primera igualdad determina l contando sobre cada factor m de n! los j


tales que pj divide m, luego en la segunda igualdad podemos intercambiar
las sumas ya que ambas son finitas (aunque una de ellas est indexada hasta el infinito, esta claro que es finita), la tercera igualdad resulta de contar
los nmeros entre 1 y n que son divisibles por pj , luego en la desigualdad se utiliza la propiedad de la funcin parte entera vista ms arriba y
mayoramos finalmente por una suma geomtrica.

2.1.

Funciones multiplicativas

Definicin 4. Sea f : N R una funcin aritmtica. Decimos que f es multiplicativa si f (mn) = f (m)f (n), m, n N tales que (m, n) = 1.
Observacin 3. Si f . 0 y si f es multiplicativa, entonces f (1) = 1. En
efecto, f (1) = f (1 1) = f (1)2 f (1) = 0 o f (1) = 1, la primera posibilidad
se descarta ya que f . 0 (de hecho f (1) = 0 permite mostrar, por multiplicabilidad de f , que f 0 lo que es contrario a la hiptesis).
Observacin 4. Por multiplicabilidad, es suficiente conocer f sobre las
potencias de los nmeros primos para conocer f en todas partes.

Lema 3. Sea f : N R multiplicativa. Entonces g(n) =

f (d) es tambin

d|n

multiplicativa.
Demostracin. Sean m y n primos entre si (es decir (m, n) = 1), entonces
X
X
X
(m,n)=1 X
(d,d 0 )=1 X
g(mn) =
f (d) =
f (dd 0 ) =
f (d)
f (d 0 )
d|m d 0 |n,(d,d 0 )=1

d|mn

d|m

d 0 |n

= g(m)g(n).

2.2.

La funcin de Euler

Definicin 5. Para n 1, la funcin de Euler se define por (n) := {1


j n : (j, n) = 1}.
Esta funcin es un ejemplo de funciones multiplicativas como se puede
ver fcilmente gracias al lema siguiente:
Lema 4.
(n) = n

Y
p|n

1
1
p

Demostracin. Si n = p1 1 pk k , entonces:
!
X n
Xn X n
Y
1
= n
+

+
n
1
p
p
p
p
p
p
p
r
r
s
r
s
t
r
r>s
r>s>t

(1)

p|n

Ahora bien, se puede concluir la demostracin con el principio de inclusinexclusin o bien mediante el siguiente argumento:
Observemos que pnr es el nmero de enteros positivos ms pequeos o
iguales a n divisibles por pr , entonces

X
X
X

1
(1) =
1
+
1

m=1

r,pr |n

r>s,pr ps |n

n
X

!
!
!
l(m)
l(m)
=
1
+

1
2
m=1 |
{z
}
:=b(m)

donde l(m) = |{p primo : p | n}|. Si l(m) = 0 entonces b(m) = 1 y si l(m) > 0
entonces b(m) = (1 1)l(m) = 0. Ahora bien (1) = |{m : l(m) = 0}| = (n) ya
que l(m) = 0 (m, n) = 1.
Lema 5.

(d) = n.

d|n

Demostracin. Observemos que la funcin identidad id : N N es multiplicativa y que el miembro de izquierda de la igualdad lo es tambin por
el lema 3. Entonces, por la observacin 4 sera suficiente demostrar esta
igualdad para las potencias de primos pj :
X
lema 4
(d) = 1 + (p) + (p2 ) + + (pj ) = 1 + (p 1) + + (pj pj1 ) = pj .
d|pj

2.3.

La funcin de Mbius

Definicin 6. La funcin de Mbius est definida por:

1
si n = 1

0
si p primo tal que p2 | n
(n) :=

k
(1)
si n = p p
1

Se verifica directamente de la definicin que es multiplicativa.


Lema 6.
X
d|n

(
1 si n = 1
(n) =
0 si no

Demostracin. De nuevo, por el lema 3, como es multiplicativa, el miembro a la izquierda de la igualdad de arriba es una funcin multiplicativa
(aplicada a n). De la misma forma el miembro a la derecha es una funcin
multiplicativa. Entonces ser suficiente verificar esta igualdad sobre las
potencias de nmeros primos pj .
Para j = 0 es evidente. Si j > 0 tenemos:
X
(d) = (1) + (p) = 1 1 = 0.
d|pj

10

2.4.

Producto de convolucin

Sean f , g : N C dos funciones aritmticas. El producto de convolucin


f g de f y g es la funcin aritmtica definida por la frmula:
 
X
n
(f g)(n) :=
f (d)g
.
d
d|n

Lema 7. Para todas f , g, h funciones aritmticas se verifica:


(i) f g = g f
(ii) (f g) h = f (g h)
(iii) f (g + h) = f g + f h
Demostracin.
  c:= n X
X
n
=d
(i) (f g)(n) =
f (d)g
f (d)g(c), como esta ltima expresin es
d
dc=n

d|n

simtrica en los divisores de n, se obtendr similar expresin para (gf )(n).


X
X  n  c:= n 0 X
(ii) f (g h)(n) =
h
f (d)
=dd
f (d)g(d 0 )h(c), luego conclui0
dd
0
0 n
d|n

dd c=n

d |d

mos como en (ii) por un argumento de simetra.


(iii) Trivial.
Proposicin 3. Si f y g son multiplicativas, entonces f g es multiplicativa.
Demostracin. Sean m y n tales que (m, n) = 1. Observemos que si d | m,
d 0 | n entonces (d, d 0 ) = 1. Adems todo c tal que c | mn es de la forma
c = dd 0 con d | m y d 0 | n. Luego
 XX



X
mn
mn
(f g)(mn) =
f (d)g
=
f (d 0 d 00 )g 00 0 .
d
d d
0
00
d|mn

d |n d |m

Donde, siguiendo la observacin de arriba, hemos escrito d = d 0 d 00 con


d 0 | n y d 00 | m. Luego por multiplicabilidad de f y g se tiene
   
XX
m
n
0
00
(f g)(mn) =
f (d )f (d )g 00 g 00
d
d
d 0 |n d 00 |m
 X
 
X
n
m
=
f (d)g 0
f (d 00 )g 00
d 00
d
0
d |n

d |m

= (f g)(n)(f g)(m).

11

Ejemplos
(1) La funcin aritmtica definida por:
(
1 n=1
(n) :=
0 sino
es una funcin multiplicativa (verificacin trivial). Si f es una funcin aritmtica cualquiera:
 
X
n
(f )(n) =
f (d)
= f (n) f = f  =  f
d
d|n

es decir,  es elemento neutro respecto a la operacin .


(2) La funcin aritmtica 1, definida por 1(n) = 1 n es multiplicativa
(de nuevo, la verificacin es trivial).
(3) La funcin aritmtica Id, definida por Id(n) = n n es multiplicativa.
Tomemos la funcin de Euler, tenemos que:
lema 5

Id(n) = n =

(n) = ( 1)(n) Id = 1

d|n

Sean a, b : N C dos funciones aritmticas (o si se prefiere, dos sucesiones


en C).
Definicin 7. Una serie de tipo

X
a(n)
n=0

ns

, se llama serie de Dirichlet.

De un curso de anlisis complejo, no es difcil ver que el dominio de convergencia absoluta de una serie de Dirichlet es un semiplano abierto de C,
limitado por una recta cuyos puntos tienen la misma abscisa. Este semiplano puede ser vaco o todo el plano complejo C.
La multiplicacin formal de dos series de Dirichlet

X a(n) X
b(n) X c(n)

=

ns
ns
ns
n=1

n=1

es otra serie de Dirichlet, donde c(n) =

n=1

X
cd=n

12

a(c)b(d) = (a b)(n).

Proposicin 4.  = 1
Demostracin. Como , y 1 son multiplicativas y por el lema 3, ser suficiente verificar esta igualdad para las potencias de primos pj , j 1:
X
( 1)(pj ) =
(pi ) 1(pji ) = (1) + (p) + + (pj ) = 0 = (pj ).
| {z }
0ij
=1

Observacin 5. Si f es una funcin aritmtica cualquiera, entonces


 
X
X
X
n
g(n) :=
f (d) =
f (d) 1 =
f (d)1
= (f 1)(n).
d
d|n

d|n

d|n

Proposicin 5. (frmula
Pde inversin de Mbius)
Sean f : N C y g(n) = d|n f (d), entonces
 
X
n
.
f (n) =
(d)g
d
d|n

Demostracin. Por la observacin 5 tenemos que g = f 1. Si convolucionamos ambos miembros de esta igualdad por tenemos: g = (f 1) =
prop 4

f (1 ) = f ( 1) = f  = f .

2.5.

Las funciones y
(n) := |{d N : d | n}|
X
(n) :=
d
d|n

Por ejemplo (6) = 4 ya que los divisores de 6 son 1,2,3 y 6. Del mismo
modo (6) = 1 + 2 + 3 + 6 = 12.
Propiedades
(1) (p) = 2 p es primo.
(2) (p) = 1 + p p es primo.
P
P
(3) (n) = (1 1)(n) = d|n 1 y (n) = (Id 1)(n) = d|n d. Es decir y
son funciones multiplicativas (proposicin 3).
13

Por las propiedades precedentes, podemos calcular fcilmente el valor de


:
j +1
k
Y
pi i 1
j1
jk
j1
jk
n = p1 pk (n) = (p1 ) (pk ) =
, ya que (pj ) = 1 + p +
pi 1
i=1

pj+1 1
p2 + + pj =
si p es primo.
p1

2.6.

Nmeros perfectos

Definicin 8. Decimos que un nmero natural n es perfecto si (n) = 2n. De


manera equivalente, decimos que n es perfecto si es igual a la suma de todos sus
divisores estrictamente menores que n.
Por ejemplo, 6 es perfecto ya que 6 = 1 + 2 + 3. Tambin lo es 28, ya que
28 = 1 + 2 + 4 + 7 + 14. El siguiente nmero perfecto es 496.
Ya en los tiempos de Euclides, se saba que n = 2p1 (2p 1) es perfecto
si p y 2p 1 son primos. Los nmeros primos de la forma Nm = 2m 1
se llaman primos de Mersenne, de hecho se puede mostrar que si Nm
es primo, entonces m es necesariamente primo. En efecto, si m no fuese primo, entonces m = ab con a, b > 1, luego 2m 1 = 2ab 1 = (2a
1) 1 + 2a + 22a + + 2(b1)a con cada uno de los factores entre parntesis
estrictamente ms grandes que 1, lo que contradice el hecho que Nm sea
primo.
Existen muchas preguntas sobre los nmeros perfectos que an no se han
resuelto, como por ejemplo: Un nmero prefecto puede ser impar? Existe
un nmero infinito de nmeros perfectos?
Teorema 3. (Euclides-Euler) Sea n par.
n es perfecto n = 2p1 (2p 1) donde p y 2p 1 son primos.
Demostracin. (Euclides):
Sea n = 2p1 (2p 1), pongamos q = 2p 1. Tenemos que (n) = 1+2+22 + +
2p 1
(1 + 2p 1) =
2p 1+q +2q + +2p1 q = (1+2+22 + +2p1 )(1+q) =
22
2 2p1 (2p 1) = 2n.
(Euler):
Por hiptesis general, podemos escribir n = 2k m, con k > 0 y m impar.
14

Tenemos que (2k m) = (2k ) (m) = (2k+1 1) (m). Por hiptesis n es perfecto, es decir (n) = 2n, luego 2n = 2k+1 m = (2k+1 1) (m) (m) = 2k+1 l,
m
. Distinguimos 2 casos:
donde l := k+1
2 1
l>1
En tal situacin 1, l, m son algunos divisores distintos de m, por tanto
(m) l + m + 1 pero l + m = 2k+1 l = (m) contradiccin!
l=1
(m) = 2k+1 = (2k+1 1) +1 = m + 1 m es primo. Ahora bien m es
| {z }
=m

un primo de Mersenne, por lo tanto k + 1 es necesariamente primo,


digamos p = k + 1. Finalmente n = 2k m = 2p1 (2p 1)

2.7.

Funcin zeta de Riemann

La funcin zeta de Riemann, denotada por , es la serie de Dirichlet asociada a la sucesin constante a(n) = 1, es decir
(s) :=

X
1
.
ns
n=1

Ella converge para Re(s) > 1.


Teorema 4. (una frmula de producto de Euler)
Y
(1 ps )1 , cuando Re(s) > 1.
(s) =
p primo

Demostracin. Sabemos que si |x| 1 entonces 1 + x + x2 + =


P 1
1
k=0 psk = 1psk si Re(s) > 1.
Observemos que
N
X
1

ns
n=1

Y
pN , primo

Y
pN , primo

!
1
1
1
1 + s + 2s + + N s
p
p
p
!
Y
1
(1 ps )1 .

s
1p
p primo

15

1
1x ,

luego

La parte derecha de esta desigualdad no depende de N . Ahora bien si


tomamos N , tenemos que la parte izquierda converge hacia (s), es
decir

X
Y
1
(s) =

(1 ps )1 .
ns
p primo

n=1

Para mostrar la otra desigualdad, observemos de nuevo que gracias al teorema fundamental de la aritmtica se tiene
Y
p primo, pN

! X

1
1
1
1 + s + + Ms
= (s).
p
ns
p
n=1

En efecto, por la unicidad de la descomposicin de un nmero en sus factores primos, al desarrollar el producto de la parte izquierda de esta de1
sigualdad se obtienen fracciones del tipo s con los m distintos.
m
Dejemos que los enteros N , M . Se tiene entonces
Y

s 1

(1 p )

p primo

X
1

= (s)
ns
n=1

Corolario 1. Existe una infinidad de nmeros primos.


Demostracin. Supongamos que existe un nmero finito de nmeros primos. Si s = 1, (1) es la serie armnica, la cual sabemos diverge.
Pero como
Q
hay un nmeros finito de primos, el producto de Euler p (1 p1 )1 es
finito, lo que contradice el teorema 4.

2.8.

Serie de ejercicios 2

1. Sea n 2 unX
entero con exactamente k factores primos distintos.
Mostrar que
|(d)| = 2k .
d|n

2. Dar una expresin de

(d) (d) en la que intervengan solamente

d|n

los factores primos de n.

16

3. Mostrar que

X (n)
nx

6
x + O(log x), donde es la indicatriz de
2

Euler.
Recordatorio: Decimos que f (x) = O(g(x)), si existen dos constantes
xa

d > 0 y C tales que x (|x a| < d |f (x)| < C|g(x)|).


4. Sea la funcin de Von Mangoldt, es decir la funcin aritmtica
definida por (n) = log p si n = pk con k 1, (n) = 0 sino. Calcular
X
X (n)
en funcin de (s) y 0 (s).
(d) y expresar
s
n
d|n

n1

17

3.

Congruencias

En la mayor parte de este captulo se tratarn temas ya vistos probablemente en cursos de lgebra 1, por lo que nos limitamos a hacer un breve
repaso de los principales resultados ya conocidos.
Definicin 9. Sean x e y dos enteros, n un nmeros natural. Decimos que
x y mod n (lase x es congruente a y mdulo n) si por definicin existe k Z
tal que x = y + kn.
Sabemos que si cocientamos el anillo Z con el ideal nZ, el resultado
2,
, n 1}, llamados clases de
es otro anillo Z/nZ cuyos elementos son {1,
restos. El ideal nZ es maximal si y solamente si n es primo, lo que implica
que Z/nZ es un cuerpo si solamente si n es primo. Si p es primo, el cuerpo
Z/pZ se escribe a menudo Fp . Para un anillo (A, +, ) cualquiera, sabemos
tambin que los elementos invertibles respecto a la operacin forman
un grupo, el que denotamos por A o bien por U (A). En particular tenemos
el grupo (Z/nZ) .

3.1.

Teorema chino del resto

Nuestro objetivo es resolver ecuaciones lineales mdulo n, n N. Por


ejemplo: encontrar un entero x tal que ax b mod n, es decir si existe x
y por ende otro entero k tales que ax = b + kn o de manera equivalente
ax + (k)n = b. Ahora bien, por la proposicin 2, sabemos que tales x y (k)
existen si y solamente si (a, n) | b. Supongamos que d = (a, n) divide b, en
ese caso la solucin x existe y se puede calcular gracias al algoritmo de
Euclides.
Un caso particular es cuando (a, n) = 1, en tal situacin a es invertible
luego levantamos un elemento x Z que
en Z/nZ y por lo tanto x = a1 b,
pertenezca a la clase de x para obtener una solucin de nuestro problema
original. Observemos que x es una solucin nica mdulo n.
Entonces para resolver la ecuacin ax b mod n es necesario y suficiente que d = (a, n) | b, en ese caso podemos dividir la ecuacin ax + (k)n = b
por d para obtener a0 x + (k)n0 = b0 , donde a0 = na , n0 = dn y b0 = nb . Entonces
(a0 , n0 ) = 1 y el problema se reduce al caso particular.

18

Teorema 5. (Teorema chino del resto) Sean n1 , , nk tales que (ni , nj ) = 1


si i , j. Sean ci , , ck Z. Entonces, existe una solucin x del sistema:

x c1 mod n1

x c2 mod n2

..

x ck mod nk
Adems, esta solucin es nica mdulo n1 nk .
Demostracin. Existencia:
Sean n = n1 nk , Mi = nni para i = 1, ..., k. Por hiptesis (ni , Mi ) = 1
yi Z tal que yi Mi = 1 mod ni . Pongamos x = y1 M1 c1 + + yk Mk ck , no
es difcil ver que tal x satisface x ci mod ni para i = 1, ..., k.
Unicidad:
Supongamos que existe otro y tal que y ci x mod ni para i = 1, ..., k,
entonces ni | y x i n1 nk = n | y x ya que los nj son primos entre
si dos a dos, luego n | y x y x mod n.
Otra forma de presentar este resultado es, bajo las mismas hiptesis, la
existencia de un isomorfismo de anillos entre Z/ (n1 Z nk Z) y Z/n1 Z
Z/nk Z.

3.2.

Teoremas de Fermat y Euler

Por definicin, tenemos que


(n) = |{ elementos inversibles de Z/nZ}| = (Z/nZ) .
En efecto, (n) cuenta todos los elementos que son primos relativos con n,
es decir todos los elementos inversibles de Z/nZ.
Proposicin 6. (Euler 1760)
Si (a, n) = 1, entonces a(n) 1 mod n.
Demostracin. (a, n) = 1 a (Z/nZ) que es un grupo de orden (n),
luego por el teorema de Lagrange de la teora de grupos, el orden de a divide
el orden del grupo, es decir a(n) = 1 a(n) 1 mod n.
Esta prueba no es la dada por Euler ya que es moderna para su poca,
sin embargo es en esencia la misma. Un caso particular de esta proposicin
es el llamado:
19

Proposicin 7. Pequeo teorema de Fermat (1640)


Sea p primo y a N, entonces ap a mod p.
Demostracin. si a 0 mod p, es trivial. Sino, a es invertible en Z/pZ, es
decir (a, p) = 1 luego por la proposicin 6, a(p) = ap1 1 mod p.

3.3.

Teorema de Wilson

Proposicin 8. n es primo (n 1)! 1 mod p.


Demostracin. )
Supongamos que n > 2 (para n = 2 la proposicin se verifica fcilmente) es
primo. Para 0 < a < p, a1 existe mdulo n.
a = a1 mdulo n a2 = 1 mdulo n a = 1, n1. Los elementos restantes
{2, 3, ..., n 2} pueden ser agrupados en parejas, cada pareja consiste en un
nmero y su inverso en Z/nZ. Entonces tenemos que:
2 3 4 n 2 1 mod n 1 2 (n 2)(n 1) (n 1) mod n
)
Supongamos por el absurdo que n = ab, con 1 < a, b < n 1.
Si a , b, entonces ab | (n 1)! (n 1)! 0 mod n lo que contradice la
hiptesis.
Si n = a2 2a < n 1, luego n = a2 | (n 1)! y de nuevo se contradice
la hiptesis.

3.4.

Teorema de Lagrange

Consideremos un polinomio con coeficientes enteros f Z[X]. Estamos


interesados en la siguiente ecuacin modular
f (x) 0 mod n.
Definicin 10. El grado mdulo n deg (f ) de un polinomio con coeficientes
enteros es el m ms grande tal que f (x) = am xm + am1 xm1 + + a0 , am .
0 mod n.
Contrariamente a lo que ocurre en un cuerpo, la ecuacin f (x) 0 (n) puede admitir un nmero de soluciones superior al de su grado mdulo n o
20

tambin no poseer ninguna. Por ejemplo, la ecuacin 2x 4 (6) posee dos


soluciones x = 2 y x = 5. Por el contrario, la ecuacin 2x 5 (6) no posee
ninguna ya que (2, 6) = 2 - 5 (ver teorema chino del resto).
Ejemplos
(1) Observacin: Si f (x) 0 mod n y que d | n, entonces f (x) 0 mod n.
Consideremos la ecuacin f (x) = x2 + x + 7 0 mod 130. Como en
principio no tenemos un mtodo general para resolver una tal ecuacin, podramos ensayar con todos los posibles valores de Z/130Z,
lo cual no es una buena idea. Pero si podemos intentar mostrar que
esta ecuacin no posee ninguna solucin. En efecto, como 2 | 130 y
ya que en Z/2Z slo tenemos dos elementos {0, 1}, es fcil ver que
f (x) . 0 mod 2, entonces por la observacin esta ecuacin no posee
soluciones en el anillo Z/130Z.
(2) Resolver f (x) = x2 + x + 7 mod 189 utilizando que f (x) 0 mod 27
tiene como soluciones A = {4, 13, 12} y que f (x) 0 mod 7 tiene como
soluciones B = {0, 6}.
Solucin: 189 = 33 7, busquemos la solucin del sistema
(
x a1 (27) a1 A
x a2 (7) a2 B
Como 27 y 7 son primos relativos, es decir (27, 7) = 1, el teorema
chino del resto se aplica. Para cada a1 A, a2 B tenemos la solucin
nica mdulo 189 dada por x = a1 b1 7 + a2 b2 27, donde b1 y b2 son
elegidos tales que b1 7 1 (27) y b2 27 1 (7), por ejemplo b2 = 4,
b1 = 1 (si no se puede adivinar fcilmente estas soluciones, siempre
es posible recurrir al algoritmo de Euclides). Las soluciones (en principio 6) son de la forma: x = a1 28 a2 27, de hecho son exactamente
seis x = 13, 49, 76, 112, 139, 175.
Este tipo de fenmenos no se producen cuando la ecuacin polinomial es tratada en un cuerpo, ese es el enunciado principal del
teorema de Lagrange.
Teorema 6. (Lagrange) Sea p primo, P Fp [X] (o bien con coeficientes en un
cuerpo cualquiera), deg (P ) = d. La ecuacin P (x) 0 mod p posee a lo mucho
d soluciones.
21

Demostracin. Por induccin en d :


d = 0 P = a0 . 0 (p), es decir sin soluciones.
d = 1 P (X) = a1 X + a0 , con a1 . 0 (p). Ahora bien, esta ecuacin no
tiene solucin si (a1 , p) - a0 y posee una solucin si (a1 , p) | a0 , adems esta
es nica (mdulo p) ya que a1 es invertible en Fp .
Supongamos que el teorema se verifica para todo polinomio de grado mdulo d inferior o igual a k. Sea P Fp un polinomio de deg(P ) = k + 1.
Supongamos adems que P posee una raz a (caso contrario no hay nada
que discutir), tenemos que:
P (X) = P (X) P (a) =

k+1
X

ai (X i ai )

i=0

k+1
X

ai (X a)(X i1 + X i2 a + + ai1 )

i=0

= (X a)G(X).
Luego, como deg(G) = k, por hiptesis de recurrencia G tiene a lo sumo k
races. Al final, P tiene a lo sumo k + 1 races.
As por ejemplo, X p 1 tiene p 1 races en Fp . Por Fermat ap1 1 (p)
a {1, 2, 3, , p 1}, es decir exactamente p 1 races y todas diferentes.
Este resultado nos permite dar otra demostracin del teorema de Wilson:
X p1 1 = (X 1)(X 2) (X (p 1)) = X p1 + + (1)p1 (p 1)!
De nuevo consideremos p > 2, es decir un primo impar (el caso p = 2 se
verifica fcilmente), tenemos que (1)p1 = 1, luego comparando los coeficientes constantes: 1 (p 1)! (p).

3.5.

Races Primitivas

Definicin 11. Sean a, n N tales que (a, n) = 1. El nmero d > 0 ms pequeo


que satisface ad 1 (1) se llama orden de a (mdulo n).
Observemos que el teorema 6 (Euler) implica que d | (n). En efecto (a, n) =
1 1 a(n) (n) aqd+r ar (n), 0 r < d r = 0 por minimalidad de d,
luego (n) = qd d | (n).

22

Pensando en trminos de grupos, a (Z/nZ) que es un grupo de orden


(n). El subgrupo engendrado por a es de orden d, luego el teorema de
Lagrange de la teora de grupos dice que el orden de ese subgrupo debe
dividir el orden del grupo.
Definicin 12. a es primitivo si su orden (mdulo n) es (n), es decir si a es
de orden maximal. En trminos de grupos, podemos re formular esta definicin
como sigue: a es primitivo a engendra (Z/nZ) .
Lema 8. Si a es de orden d (mdulo n), am es de orden

d
(d,m)

(mdulo n).

Demostracin. Por divisin euclidiana es fcil ver que ak 1 (n) d | k.


d
m
d
Sea j el orden de am , (am )j 1 (n) d | mj (d,m)
| (d,m)
j (d,m)
| j ya que
d
(d,m)

m
(d,m)

son primos relativos. Entonces

de j se tiene que

d
(d,m)

d
(d,m)

j, pero por minimalidad

= j.

Teorema 7. Sea p un primo impar. El nmero de elementos de orden d en Fp


es (d), en particular existen primitivos ya que (p 1) > 0.
Demostracin. Por definicin de orden (mdulo p), podemos particionar
los elementos de Fp = {1, 2, , p 1} = Ad1 t Ad2 t t Adk , donde di son los
divisores de (p) = p 1 y Adi = {a Fp : a es de orden di }.
X
Sea (d) = |Ad |,
(d) = p 1. Supongamos que existe d (o bien di )
d|pq

tal que (d) , 0, sea entonces a Ad .


Afirmacin: Los elementos de Ad son exactamente todos los am con 1
m d, (m, d) = 1.
En efecto, todos los am con 1 m d son diferentes (por definicin de
orden de a), adems (am )d (ad )m 1 (p), es decir los am son races de la
ecuacin X d 1 = 0 en Fp , por el teorema 6 (Lagrange) esas son todas las
races. As, todos los elementos de Ad son de la forma am y por el lema 8,
am es de orden d si y solamente si (d, m) = 1.
La afirmacin nos dice en particular que (d) = (d), si suponemos Ad ,
. Regresando al teorema, tenemos que
X
X
p1 =
(d) =
(d).
d|p1,(d),0

d|p1,(d),0

23

Como 1 = Id

(d) = p1, luego no es posible que para un divisor

d|p1

d de p1 se tenga (d) = 0. Entonces (d) , 0 d | p1 (d) = (d) d |


p 1.

3.6.

Indices (Logaritmos discretos)

Siguiendo la analoga con los nmeros reales, decimos que x es el logaritmo discreto de c en base en a mdulo n si por definicin ax c mod n.
Retomando notaciones familiares, escribiremos x = loga c o bien x = ind(c)
sobreentendiendo la base a del logaritmo.
El clculo de logaritmos discretos para n = p primo muy grande es un problema difcil, slo se conocen algoritmos subexponenciales como el Index
Calculus, es por eso que los logaritmos discretos juegan un papel importante en la criptografa.
Ejemplos
(1) Trabajemos mdulo p, p primo. El logaritmo discreto de 1 en base
a Fp est dado por x = p 1. En efecto, es el teorema pequeo de
Fermat (proposicin 7).
(2) Si x = 3 2x 3 (5), luego x = log2 3 = 3. Por otro lado, todas las
potencias de 4 son ya sea 1 o bien 4 mdulo 5, entonces log4 3 no
existe.
Fijemos g primitivo mdulo n. Para cada a (Z/nZ) existe l tal que g l
0
a mod n, es decir l = logg a. Si otro l 0 satisface g l a mod n, entonces l y
l 0 son congruentes mdulo (n). En adelante no escribiremos la base g del
logaritmo dndola por sobreentendida.
Proposicin 9. Sean a, b (Z/nZ) , g un primitivo fijado.
(i) log 1 = 0
(ii) log g = 1
(iii) log a + log b = log(ab) mod (n)
Demostracin. (iii) Como g es primitivo g (n) 1 (n) g log(ab) = ab =
g log a g log b = g log a+log b log a + log b = log(a) mod (n).

24

3.7.

Esquema de Diffie-Helman

Supongamos que Marco y Dani quieren intercambiar una clave para


poderla utilizar en un protocolo criptogrfico cualquiera. Ellos se ponen de
acuerdo en un nmero primo p y un elemento g Fp que hacen pblico.
Marco elige su clave secreta x Fp , del mismo modo Dani elige la suya
y Fp . Marco calcula con su clave secreta X g x mod p, igualmente Dani
calcula Y g y mod p. Marco le enva X a Dani y Dani le enva Y a Marco
a fin de poder intercambiarse una clave que cada uno calcula con su clave secreta. Marco descifra la clave calculando K Y x mod p e igualmente
Dani descifra la clave calculando K 0 X y mod p.
Marco y Dani tiene la misma clave ya que
K Y x (g y )x (g x )y X y K 0 modp
El espa Roberto observa lo que sucede (Roberto conoce p, g, X e Y ), l
no puede obtener la clave intercambiada entre Marco y Dani ya que no es
capaz de calcular logaritmos discretos en un tiempo razonable (puesto que
para obtener K o K 0 Roberto necesita conocer las claves secretas de Marco
o de Dani).

3.8.

Serie de ejercicios 3

1. Mostrar que

X
n=1

(n)

x
xn
=
.
n
1x
(1 x)2

2. Encontrar todos los enteros x tales que 2x 1(mod 3), 3x 1(mod 5)


y 5x 1(mod 7).
3. Mostrar que para todos p primo, a, k 1 enteros se tiene akpk+1
a(mod p). Deducir que 798 | a19 a.
4. Encontrar enteros a, b, c, d y e tales que para todo x entero, al menos
una de las congruencias x a (mod 2), x b (mod 3), x c (mod 4),
x d (mod 6) y x e (mod 12) sea satisfecha.

25

4.
4.1.

Residuos cuadrticos
Smbolo de Legendre

Hace alrededor de 4000 aos, los matemticos como Al-Khwarizmi,


saban cmo resolver las ecuaciones de tipo ax2 + bx + c = 0 por medio de
tcnicas que ellos llamaban Al-jabr, Al-muqabala que podemos traducir
por balanceo y complementacin. Cuando se trabaja en un cuerpo diferente de R, como por ejemplo Z/pZ, esta ecuacin presenta la dificultad
siguiente: es = b2 4ac un cuadrado mdulo p? Suponiendo que p no
divide 4a.
Nos interesamos entonces en la ecuacin modular x2 k mod p. Observemos que:
Si p = 2 x = 0 o x = 1 (la ecuacin siempre tiene solucin).
Si k = 0 y p es un primo cualquiera, x = 0 es la nica solucin.
Si x es una solucin, entonces x lo es tambin.
Para p , 2 se tiene que x . x (p) y entonces, si conocemos una solucin x, por el teorema 6 (Lagrange) conocemos todas las soluciones
ya que la ecuacin posee a lo sumo dos races.
Definicin 13. Sean a y n tales que (a, n) = 1. Decimos que a es un residuo
cuadrtico mdulo n si la ecuacin x2 a (n) posee una solucin.
Definicin 14. Sea p primo, el smbolo de Legendre se define por:

!
0
si p | a

1 si a x2 (p)
=

p
1
si no
Ejemplos
 
a
(1)
= 1 ya que 02 = 0 y 12 = 1.
2
!
1
(2)
= 1 ya que (1)2 = 1.
p

26

4.2.

Criterio de Euler
p1
2

residuos cuadrticos en Fp . Ade p1 2


ms, estos residuos cuadrticos son 12 , 22 , ..., 2 .
Proposicin 10. Sea p > 2 primo. Hay

 p1 2
p1
Demostracin. Pongamos r = 2 . Claramente 12 , 22 , ..., 2
son residuos
cuadrticos, adems son todos diferentes. En efecto, supongamos que a2
b2 (p) (a + b)(a b) 0 (p), luego como Z/pZ es un cuerpo, en particular
un dominio integral, se tiene a + b 0 o bien a b 0. Si a + b 0, tenemos
p1
que a + b = p a = p b > r para b {1, ..., 2 }, luego a = b.
p1

Mostremos ahora que {1, ..., 2 } son todos los residuos cuadrticos. Cada
vez que y x2 (p), por el teorema de Fermat (teorema 7): y r x2r xp1
1 (p), luego los residuos cuadrticos satisfacen la ecuacin y r 1 0 (p) y
p1
por el teorema de Lagrange (teorema 6) existe a lo sumo 2 soluciones a
dicha ecuacin, las cuales conocemos todas.
Observacin 6. Si g es un elemento primitivo, entonces 1, g 2 , g 4 , ..., g 2(r1)
es la lista de residuos cuadrticos.
Proposicin 11. (Euler) Si p > 2 primo, entonces:
!
 p1 
a
a 2 mod p.
p
Demostracin. Si a = 0 la igualdad es verificada. Pongamos como antes
p1
r = 2 y distingamos dos casos:
!
a
Fermat
(1) Si
= 1 existe x tal que a x2 (p), luego ar x2r xp1 1 (p).
p
!
a
(2) Si
= 1 se tiene que ar . 1 (p). Por Fermat 1 ap1 (ar )2 ar
p
1 (p), luego ar 1 (p).

!
! !
ab
a b
Corolario 2. (i)
=
p
p p
!
!
a
b
=
(ii) a b (p)
p
p
27

!
p1
1
= (1) 2
(iii)
p
Observacin 7. No hay necesidad de considerar estas relaciones mdulo
p.

4.3.

Lema de Gauss

Definicin 15. El representante ms pequeo de a mod n es el a0 tal que


a0 a mod n con 21 n < a0 12 n.
Ejemplo
2 es el representante ms pequeo de 15 mdulo 17.
Lema 9. (Gauss) Sea p > 2 primo y a tal que (a, p) = 1. Sean aj los represenp1
tantes ms pequeos de j a, para j = 1, 2, ..., 2 . Entonces:
!
a
= (1)l ,
p
donde l = |{1 j

p1
2

| aj < 0}|.

Ejemplo p = 7, a = 2. Los representantes ms pequeos


  modulo 7 de j a
2
para j = 1, 2, 3 son 2, 3 y 1, luego por el lema 9:
= (1)2 = 1. Por
7
ejemplo 42 2 (7).
Demostracin. Afirmacin: {|aj |}rj=1 = {1, 2, ..., r}.
Prueba de la afirmacin: Por definicin 1 |aj | r. Mostremos que son todos diferentes:
a,0
Caso 1: aj = ak ak +aj = 0 a(j +k) = 0 k +j = 0 lo que no es posible
ya que 0 < k + j p.
a,0

Caso 2: aj = ak a(j k) = 0 j = k.
De regreso al lema, gracias a la afirmacin |a1 | |ar | = r! a1 ar =
r!(1)l . Sabemos tambin que aj ja (p), luego a1 ar r!ar (p). Comparando estas dos igualdades mdulo p tenemos ar ! (1)l (p), finalmente
a
ar (1)l (p).
por el criterio de Euler (proposicin 11) tenemos
p
Corolario 3.
!
1 2
2
= (1) 8 (p 1) .
p
28

4.4.

Ley de reciprocidad cuadrtica

Teorema 8. (Gauss 1976) Sean p, q dos primos diferentes e impares, entonces:


! !
p1 q1
p q
= (1) 2 2 .
q p
Este teorema, a veces llamado "teorema ureo" permite relacionar la solubilidad de dos congruencias de segundo grado: x2 p mod q, y 2 q mod p.
En efecto la frmula de reciprocidad cuadrtica nos permite ver cundo
p y q son o no simultneamente cuadrados mdulo q y p respectivamente. Se conoce alrededor de 200 demostraciones diferentes de esta frmula,
nosotros examinaremos una de ellas que es bastante simple y original.
Demostracin. La idea principal de esta demostracin es la de utilizar el
lema 9. Con una notacin ligeramente diferente, pongamos j = x y aj =
px qy donde y se determina de las relaciones 12 q < aj , 0 < x < 12 q, es
px
decir y < q + 21 < 21 (p + 1). Como y es entero, el lema 9 puede reformularse
como sigue:
!
p
= (1)l
q
l = |{(x, y) Z2 | 0 < x < 12 q, 0 < y < 21 p tales que 12 q < px qy < 0}|.
De manera completamente simtrica:
!
q
= (1)m
p
m = |{(x, y) Z2 | 0 < x < 12 q, 0 < y < 12 p tales que 12 p < qy px < 0}|.
h
i h
i
Sobre el rectngulo D = 0, 21 q 0, 12 p de la figura 1. Las rectas px qy =
0, pxqy+ 21 q = 0 y px+qy+ 21 p no intersecan ningn punto en el interior de
T
la red Z2 D. Los puntos enteros (x0 , y 0 ) dentro del tringulo superior del
rectngulo D est en biyeccin con los puntos enteros (x, y) del tringulo
inferior del rectngulo D va las siguientes frmulas:
1
1
x = (q + 1) x0 , y = (p + 1) y 0
2
2

 
 p q
q
Por ejemplo px0 qy 0 = p 12 (q + 1) x q 21 (p + 1) y = 2 2 px+qy 2 .
29

Figura 1: Reciprodidad cuadrtica


p1 q1
Finalmente, hemos mostrado que

(l + m) es par, luego
2
2
! !
p1 q1
p q
= (1)l+m = (1) 2 2
q p
Ejemplo. Utilizando los corolarios 2 y 3 y el la ley de reciprocidad cuadrtica, mostrar que -42 es un cuadrado mdulo 61.
Solucin:

       
42
1
2
3
7
=
61
61 61 61 61
|{z} |{z}
1

611
1
2
1
= (1) 2 = 1 y
= (1) 8 (62)(60) = 1. Continuando de la
61
61
misma manera
   
   
 
611 71
71 51
7
61
5
7
2
=
(1) 2 2 =
=
(1) 2 2 =
61
7
7
5
5

ya que

2 1)

= (1) 8 (5

= 1.

y


 
 
3
61
1
=
(1)301 =
= 1.
61
3
3


42
Finalmente
= 1 (1) (1) 1 = 1.
61


30

4.5.

Serie de ejercicios 4

1. Encontrar todos los enteros x tales que 3x 1(mod 5), 5x 1(mod 17)
y 7x 1 (mod 23).
2. Sean a1 , ..., ap y b1 , ..., bp dos sistemas completos de residuos mdulo
p, con p primo (un conjunto X = {x1 , ..., xn } de n enteros es un sistema
completo de residuos mdulo n si para todo entero x Z existe un nico
xk X tal que xk x (mod n)). Mostrar que a1 b1 , ..., ap bp no puede ser
nunca un sistema completo de residuos mdulo p si p , 2.
3. Mostrar que, para p 3 primo,
la congruencia x2 (1)

p1
admite por solucin x = 2 !.

p+1
2

(mod p)

4. Sean a, n 2 dos enteros tales que an1 1(mod n) pero am . 1(mod n)


para todo divisor m , n 1 de n 1. Mostrar que n es primo.

4.6.

Smbolo de Jacobi

Una generalizacin del smbolo de Legendre es el smbolo de Jacobi.


Sean n = p1 pk un nmero impar con los pi primos (no necesariamente
diferentes) y a un entero, el smbolo de Jacobi se define por la siguiente
frmula:
Qk  a 
 

i=1 pi si (a, n) = 1
a

:=
1
si n = 1

0
si (a, n) > 1
!
a
donde
es el smbolo de Legendre.
pi
Si n = p primo, el smbolo de Jacobi no es otra cosa que el smbolo de
Legendre, una razn por la cual no utilizamos una nueva notacin.
Observacin 8. Si a es un
 cuadrado mdulo n, es decir existe x tal que
a
2
x a mod n, entonces
= 1. Pero contrariamente a lo que nos dice por
n
definicin el smbolo de Legendre, si n no es primo, la afirmacin recproca para el smbolo de Jacobi, es en general falsa.
    
2
2 2
Ejemplo.
=
= (1)(1) = 1, sin embargo un clculo a la mano
15
3 5
de los restos cuadrticos mdulo 15 nos muestra que x2 2 mod 15 no
tiene solucin.

31


(i)

Proposicin 12.

   
a a
a
=
mn
m n

!   !
ab
a b
(ii)
=
n
n n
 
n1
1
(iii)
= (1) 2
n
 
n2 1
2
(iv)
= (1) 8
n
m
(v) Si (m, n) = 1, entonces
n





m1 n1
n
= (1) 2 2
m

Demostracin. Observemos que el smbolo de Jacobi generaliza las propiedades ya conocidas para el smbolo de Legendre, las tres primeras se
demuestran directamente. Para mostrar (v) procedemos como sigue:
Observemos que si a y b son impares, entonces
!
(a 1)(b 1)
a1 b1
ab 1

+
0 mod 2.
=
2
2
2
2
Q
Q
Supongamos que m = ri=1 pi y n = sj=1 qj , tenemos que


!
r Y
s
 Y
pi
m
=
por las propiedades (i) e (ii)
n
qj
i=1 j=1
!
r Y
s
Y
pi 1 qj 1
qj
=
(1) 2 2 por reciprocidad cuadrtica
pi
i=1 j=1
r Y
s
Y


=

n
m

(1)

i=1 j=1
Ps Pr

n
=
(1)
m


pi 1 qj 1
2 2

j=1

i=1

pi 1 qj 1
2 2

Ahora solo queda mostrar que


r X
s
X
pi 1 qj 1 m 1 n 1

mod 2.
2
2
2
2
i=1 j=1

32

(2)

En efecto, aplicando la frmula (2) las veces que sean necesarias:


r
r
X
pi 1 p1 p2 1 X pi 1

+
mod 2
2
2
2
i=3

i=1

p p p 1 X pi 1
1 2 3
+
mod 2
2
2
i=4

..
.
Qr

i=1 pi

mod 2,

luego

r
s
s
r X
X
pi 1 qj 1 X pi 1 X qj 1

2
2
2
2
i=1 j=1

Qr

i=1 pi

j=1

i=1

Qs

j=1 qj

m1 n1

mod 2.
2
2

 
a
es 1, a no es necesaria n
a
mente un cuadrado mdulo n. Sin embargo si
= 1, entonces a no es
n
un cuadrado mdulo n ya que a no lo es para algn primo pk que divide a
n.
Ya hemos visto que el si el smbolo de Jacobi de

4.7.

Serie de ejercicios 5

1. Determinar todas las soluciones de la congruencia y 2 5x3 (mod 7).


2. Sean p un primo y k un entero positivo. Mostrar que

p
X
n=1

nk es con-

gruente a 1 mdulo p si p 1 divide k y a 0 sino.


Indicacin: Escribir la suma utilizando un generador del grupo Fp .
3. Mostrar que

 
2
p

= (1)

p2 1
8

4. Es 15 un cuadrado mdulo 73?

33

5. Sea p = 4k + 3 un primo tal que p0 = 2p + 1 es tambin primo. Mostrar


que 2p 1 (mod p0 ). Deducir que 2251 1 no es primo.
Nota: Mn = 2n 1 se llama nmero de Mersenne. Si Mn es primo, entonces se dice que Mn es un primo de Mersenne.

34

5.

Formas cuadrticas

Definicin 16. Una forma cuadrtica q sobre R es una funcin


q(x1 , ..., xn ) =

n
n X
X

aij xi yj

i=1 j=1

con los aij Z.


Si ponemos x = (x1 , ..., xn )t y A = (aij ) Mat (Rn ), podemos escribir de manera ms compacta q(x) = xt Ax.
Decimos que q es definida positiva (respectivamente definida negativa)
si para todo x , 0, q(x) > 0 (respectivamente si para todo x , 0, q(x) < 0).
De manera similar se utilizan los trminos semi-definida positiva o negativamente, simplemente cambiando la desigualdad estricta por un o un
respectivamente. En caso de no cumplirse ninguna de esas condiciones, se
dice que la forma es indefinida.
El grupo de automorfimos de Zn , Aut(Zn ) es
SLn Z = {M Mat(Zn ) |detM = 1}.
Dos formas q y q son equivalentes si por definicin existe SLn Z tal que

q = q . En particular Im(q) = Im(q).


Sea A, A las matrices de las formas q y q respectivamente, entonces
A = t A,
de donde se tiene de forma directa que detA = detA.
El caso n = 2 nos da la forma cuadrtica f (x, y) = ax2 + bxy + cy 2 con
a, b, c Z. El discriminante d de f es por definicin d := b2 4ac. Si a , 0
tenemos que 4af (x, y) = (2ax + by)2 dy 2 . Si a, d , 0 los signos de a y d nos
ayudan a determinar si la forma es definida positiva (negativa) o no, por
ejemplo si a > 0 y d < 0 la forma es definida positiva. Si a < 0 y d < 0 la
forma es definida negativa. Si d = 0 et signo de a determina si la forma es
o no semi-definida positiva (negativa). Si d > 0 la forma es indefinida.
Asociamos a f la matriz
!
a b/2
A=
.
b/2 c
35

Vemos que det A = d/4, luego el signo del determinante de A est relacionado con la positividad de f .

5.1.

Suma de cuadrados

La teora aditiva de nmeros es una rama de la teora de nmeros que estudia formas de representar nmeros enteros positivos como sumas.
Si S N, podemos preguntarnos qu subconjunto de N podemos representar si sumamos los elementos de S entre si. Por ejemplo, si nos permitimos sumar nicamente dos elementos de S a la vez, la pregunta es a qu
corresponde S + S?
Ejemplos famosos
(1) Si S = {02 , 12 , 22 , 32 , ...}, Bachet conjetur en 1621 que S +S +S +S = N,
es decir que todo nmero natural se escribe como la suma de cuatro
cuadrados. Lagrange demuestra esta conjetura en 1770.
(2) Si S = {2, 3, 5, 7, ...} (los primos), Goldbach conjetur en 1742 que todo
nmero par 4 se escribe como la suma de dos primos. En nuestra
notacin S + S = { todos los pares 4}. Esta conjetura sigue sin demostrar, tampoco se la ha podido refutar ya que se ha podido comprobar por ordenador para pares muy grandes.
(3) Sigamos con S = {todos los primos}. Vinogradov conjetura que S +
S + S = {todos los impares 7} y da una prueba incompleta demostrando que es el caso para impares muy grandes, sin ser preciso
en lo que se entiende por muy grande. En el 2013, Harald Helfgott,
un matemtico peruano, publica una demostracin que actualmente
est en curso de validacin.
(4) Problema de Waring (1770): dados k N, Sk = {0k , 1k , 2k , 3k , ...} existe
un m tal que todo nmero natural se escribe como una suma de m
trminos de Sk ? En nuestra notacin Sk + + Sk = N?
| {z }
suma finita
k

Teorema 9. (Fermat-Euler) Sea n = 2 p11 ps s q11 qrr con pi , qj primos


diferentes tales que pi 3 mod 4 i y qj 1 mod 4 j. Entonces, n es suma de
dos cuadrados si y slo si todos los ki son pares.

36

Demostracin. )
Si n = x2 +y 2 con x, y Z y sea p primo tal que p | n con p 3 mod 4. Entonces 0 x2 +y 2 mod p x2 y 2 mod p. Podemos suponer que ya sea x o y
no son divisibles por p, si no no hay nada que mostrar. Sin prdida de gene!2
x
1 mod p. Pero si p 3 mod 4,
ralidad supongamos que p - y, luego
y
!
p1
1
-1 no es un cuadrado mdulo p ya que
= (1) 2 = 1, luego necep
!2
!2
y
x
n
+
p2 | n. Repetimos este
sariamente x y 0 mod p 2 =
p
p
p
argumento para todos los primos p tales que p | n con p 3 mod 4.
)
Observemos que (a2 + b2 )(c2 + d 2 ) = (ac bd)2 + (ad bc)2 (poner por ejemplo z1 = a + ib, z2 = c + id y utilizar la identidad |z1 |2 |z2 |2 = |z1 z2 |2 ). Esta
identidad es en particular cierta para todo a, b, c y d enteros, ella nos dice que si m y n son dos sumas de cuadrados, su producto mn lo es tambin.
Ahora bien, 2 = 12 + 12 y los pi tales que pi | n con pi 3 mod 4 aparek
s
cen, por hiptesis, en potencias pares ki = 2si y luego pi i = (pi i )2 + 02 . As,
slo queda por mostrar que los primos q tales que q 1 mod 4 se escriben
como la suma de dos cuadrados q = x2 + y 2 .
!
q1
1
= (1) 2 = 1 ya que q 1 (4), existe entonces x tal que 1 x2 (q).
q
h i
Consideremos la funcin f (u, v) = u+xv y sea k = q . Observemos quek <

q < k + 1 ya que q es primo. Consideremos las parejas (u, v) con 0 u, v

k, tenemos en total (k + 1)2 de tales, luego (k + 1)2 > ( q)2 = q. Si consideramos la funcin f a valores en Fq , es decir a la pareja (u, v) le asociamos el valor u + xv mod q, el principio de Dirichlet nos dice que existen
(u 0 , v 0 ) y (u, v) con 0 u 0 , v 0 , u, v k y (u 0 , v 0 ) , (u, v) tales que f (u, v)
f (u 0 , v 0 ) mod q, es decir u +xv u 0 +xv 0 mod q u u 0 x(v 0 v) mod q
(u u 0 )2 x2 (v 0 v)2 mod q (u u 0 )2 + (v 0 v)2 mod q.
Pongamos a = u u 0 , b = v 0 v, tenemos entonces a2 + b2 0 mod q, es

decir q | a2 + b2 , pero como (u, v) , (u 0 , v 0 ) y a = u u 0 k < q (de igual


forma para b) tenemos 0 < a2 + b2 < 2q, por consecuencia q = a2 + b2 .
37

5.2.

Serie de ejercicios 6

1. Determinar los primos p para los cuales 5 es un cuadrado mdulo p.


 
2. Calcular el smbolo de Jacobi 123
917 .
3. Sea p = 2n + 1 un primo. Mostrar que g es una raz primitiva mdulo
p si y solamente si, g no es un cuadrado mdulo p. Deducir que si
n > 1, 3 es una raz primitiva mdulo p.
4. Sea f (x) = ax2 + bx + c con a, b y c enteros y sea p un primo impar
que no divide a a. Mostrar que el el nmero de soluciones de la congruencia f (x) 0 (mod p) es 1 + dp , donde d = a2 4ac.
5. Mostrar que la asercin todo nmero entero superior o igual a 6
puede escribirse como la suma de tres nmeros primos es equivalente la conjetura de Goldbach.

5.3.

Sumas de cuatro cuadrados

Recordemos que Bachet enunci en 1621 su famosa conjetura: todo


nmero natural se puede escribir como la suma de cuatro cuadrados, la
cual fue demostrada ms tarde en 1770 por Lagrange. Lagrange haba incluso mostrado que no era posible hacer algo mejor, que por ejemplo 3
cuadrados no eran suficientes: 7 = 22 + 12 + 12 + 12 . De manera ms general
si n 7 (8), entonces n no puede escribirse como la suma de tres cuadrados ya que, en particular, se tendra n x12 + x22 + x32 (8), pero los cuadrados
mod 8 son x2 0, 1, 4 (8), luego combinando estos restos cuadrticos en
tres sumandos obtenemos 0,1,2,3,4,5,6 pero no 7.
Para demostrar la conjetura de Bachet, necesitaremos una identidad similar a la que utilizamos para probar el teorema 9, dicha identidad la
obtenemos de los cuaterniones H = {x + iy + jz + kw | x, y, z, w R} con i, j y k
verificando las relaciones i 2 = j 2 = k 2 = 1 y ijk = 1, a partir de las cuales
podemos deducir que ij = k y ji = k, de donde vemos que con la estructura de grupo multiplicativo H no es conmutativo. Como para los nmeros
complejos, es posible definir
p una especie de valor absoluto sobre H poniendo |x + iy + jz + kw| := x2 + y 2 + z2 + w2 . Un clculo elemental permite
ver que este valor absoluto es multiplicativo, es decir si a, b H entonces
|a||b| = |ab|, de donde |a|2 |b|2 = |ab|2 . Esta ltima relacin sera nuestro punto
de partida para demostrar el enunciado de Bachet.

38

De forma ms explcita pongamos a = x+iy +jz+kw y b = x0 +iy 0 +jz0 +kw0 ,


entonces
(x2 + y 2 + z2 + w2 )(x02 + y 02 + z02 + w02 ) = (xx0 + yy 0 + zz0 + ww0 )2 +
(xy 0 yx0 + wz0 zw0 )2 + (xz0 zx0 + yw0 wy 0 )2 + (x0 w0 wx0 + zy 0 yz0 )2 (3)
Teorema 10. (Lagrange 1770) Todo n > 0 tiene la forma n = x2 + y 2 + z2 + w2 ,
con x, y, z, w N {0}.
Demostracin. La identidad (5.3) nos dice que es suficiente demostrar es
teorema para n = p primo. Podemos incluso considerar p > 2 ya que 2 =
12 + 12 + 02 + 02 .
Sea entonces p > 2, para 0 x 21 (p 1) hemos visto por la proposicin
10 que los x2 son todos distintos. Un argumento de traslacin nos permite
concluir que de modo similar los 1 y 2 para 0 y 12 (p 1) son todos
 2
distintos, luego existen x, y tales que x2 = 1y 2 x2 +y 2 +1 < 1+2 12 p <
1 + 12 p2 < p2 .
x2 + y 2 + 1 0 mod p mp = x2 + y 2 + 1 < p2 con 0 < m < p. Podemos
escribir tambin mp = x2 + y 2 + 12 + 02 , luego vemos que se ha progresado
en el teorema y habramos terminado si m = 1. La idea ahora es de mostrar
que podemos "descender m"hasta conseguir que p se escriba como suma
de cuatro cuadrados, esta forma de proceder se conoce como el mtodo del
descenso infinito de Fermat.
Sea l > 0 el nmero entero ms pequeo tal que lp = x2 + y2 + z2 + w 2 .
Vamos a mostrar que l = 1. Sabemos de antemano que l < p puesto que
l m.
Afirmacin: l es impar.
x y,
z + w,
z w son pares
En efecto, si l es par podemos suponer que x + y,
ya que lp = par = x2 + y2 + z2 + w 2 , de donde si prdida de generalidad
vemos que hay tres posibilidades:
y deben ser pares y z, w impares.
(1) x,
y,
z, w son todos impares.
(2) x,
y,
z, w son todos pares.
(3) x,
39

luego

2 
2 
2 
2
1
1
1
1
1
+ (x y)
+ (z + w)
+ (z w)

lp = (x + y)
2
2
2
2
2
lo cual no es posible ya que l es minimal.
Sabemos ahora que l debe ser impar, supongamos que l > 1 y pongamos
n = x02 + y 02 + z02 + w02 donde x0 , y 0 , z0 , w0 son los representantes ms pe y,
z, w respectivamente mdulo l. Claramente n 0 mod l ya
queos de x,
2
que n x + y2 + z2 + w 2 = lp 0 mod l, pero n > 0 ya que si n = 0 x0 =
y,
z, w l 2 | x2 + + w 2 = lp l | p lo cual no es
y 0 = z0 = w0 = 0 l | x,
 2
posible ya que p primo. Adems n < 4 21 l = l 2 puesto que x0 , y 0 , z0 , w0 son
y,
z, w respectivamente y como l es
los representantes ms pequeos de x,
1
0
0
0
0
impar x , y , z , w < 2 l.
Como l | n y n < l 2 n = kl con 0 < k < l. La frmula (5.3) nos dice
que (kl)(lp) es suma de cuatro cuadrados: (kl)(lp) = a2 + b2 + c2 + d 2 .
Afirmacin: l 2 | a2 , b2 , c2 , d 2 .
0 + yy
0 + zz0 + ww
0 x02 + y 02 + z02 + w02 0 mod l y de
En efecto, a = xx
manera similar para b, c y d.
! 2  2
!2
 2
c
a
b
d
+
, lo que contradice de nuevo la
Finalmente kp =
+
+
l
l
l
l
minimalidad de l, sigue entonces que l = 1.

5.4.

Serie de ejercicios 7

1. Mostrar que las formas cuadrticas f (x, y) = x2 + y 2 y g(x, y) = x2 +


4xy + 5y 2 son equivalentes. Deducir la forma de los enteros que se
escriben de la forma x2 + 4xy + 5y 2 .
2. Encontrar dos formas cuadrticas f y g que no sean equivalentes
pero que representen el mismo subconjunto de enteros S Z, es decir
f (Z2 ) = g(Z2 ).
Indicacin: Utilizar el teorema de la suma de cuatro cuadrados.
3. Sea E el conjunto de los enteros que no se escriben como suma de 3

40

cuadrados. Mostrar que


1
1
lm sup |{a x : a E}| .
8
x x
4. Mostrar que ningn entero de la forma 22k+1 , con k 1, puede escribirse como suma de 4 cuadrados estrictamente positivos.

41

6.

Aproximaciones diofnticas

Consideremos las inclusiones N Z Q R C. Sabemos gracias a


nuestros primeros cursos de anlisis y lgebra que Q se obtiene de Z a partir del cuerpo de fracciones de este ltimo y que a su vez R es la complecin arquimediana de Q, finalmente aadiendo un elemento i verificando
i 2 = 1 obtenemos una extensin de R que denotamos por C. Los tres primeros conjuntos de estas inclusiones, N, Z, Q son numerables, mientras que
los dos ltimos R, C no. Si imaginamos a Q en la recta real, el hecho que
ste sea numerable (adems de ser de medida de Lebesgue nula!) y R al
contrario no lo sea, nos dice que existen agujeros entre Q y R. Para intentar
comprender qu sucede y cmo llenar esos agujeros, vamos a interesarnos
en los nmeros algebraicos.
Definicin 17. Un nmero C se llama algebraico si existe un polinomio
el conjunto de nmeros
no nulo P Q[X] tal que P () = 0. Denotaremos por Q
algebraicos sobre Q.
Observemos que en la definicin, podemos exigir de manera equivalente
un polinomio no nulo P Z[X]. Llamamos polinomio minimal de un nmero algebraico al nico polinomio de Q[X] unitario de grado minimal
que se anula en . Sea d el grado del polinomio minimal de . Diremos
que es un nmero algebraico de grado d.
Diremos que C es trascendente si no es algebraico. A finales del
siglo XIX se demostr que e (Hermite) y (Lindeman) son trascendentes.
Determinar si un nmero es o no transcendente es en general muy difcil, de hecho an no se ha podido demostrar si por ejemplo
PN 1e + es o no
e
e

trascendente, as como tambin: , e , , = lmN n=1 n log N que


an son preguntas abiertas.

6.1.

Teorema de Dirichlet

Teorema 11. (Dirichlet) Para todo R et para todo Q > 1, existen p, q Z,


0 < q < Q tales que |q p| Q1 .
Demostracin. Sea R, Q > 1. Supongamos que Q Z y consideremos
los Q + 1 nmeros siguientes
0, {}, {2}, , {(Q 1)}, 1 [0, 1]

42

Dividamos el intervalo [0, 1] en Q sub-intervalos de largo Q1 . Por el principio de Dirichlet, existen dos de esos nmeros x1 y x2 que se encuentran en
el mismo sub-intervalo.
Si x1 , x2 , 1, existen Q 1 m1 > m2 0 tales que |{m1 } {m2 }| Q1 .
Pongamos q = m1 m2 y p = [m1 ] [m2 ], tenemos que 0 < q < Q. Luego
|p q| = |(m1 m2 ) ([m1 ] [m2 ])| = |{m1 } {m2 }|

1
Q

Si uno de los xi (i = 1 o i = 2) es igual a 1, se procede de la misma manera


(en ese caso para uno de los xi no ser necesario escribir partes enteras y
partes fraccionarias).
Si Q no es entero, procedemos como antes con [Q] + 1 y luego observa1
< Q1 .
mos que [Q]+1
Observacin 9. Podemos exigir que (p, q) = 1, ya que si (p, q) = d > 1 podemos poner en evidencia el factor d del miembro izquierdo de la desigual1
< Q1 .
dad, luego dividir por d ambos lados y finalmente observar que dQ
Corolario 4. (1) Si es irracional, entonces existe una infinidad de raciop
p
nales q tales que | q | q12 , con (p, q) = 1.
(2) Si es racional, entonces existe un nmero finito de racionales
que |

p
1
q | q2 ,

p
q

tales

con (p, q) = 1.

Este corolario puede parecer contra-intuitivo ya que nos dice que los racionales no se pueden aproximar tan bien como los irracionales con sucesiones de racionales (salvo la sucesin constante igual al nmero racional
en cuestin). Este fenmeno sucede por algo que ya habamos mencionado
antes, que es el hecho que Q visto sobre R est lleno de agujeros.
Demostracin. Para no preocuparnos de problemas de signo, vamos a suponer 0. Esto no supone ninguna restriccin ya que para < 0 tendremos demostrado el corolario para y un simple cambio en el signo de p
nos llevar de vuelta al resultado que buscamos.
(1) Sea irracional y Q > 1. Por el teorema 11 existe
p

p
q

tal que | q |

1
q2

con (p, q) = 1. Como no es racional, 0 , | q | 1, luego existe Q2 > 1


tal que Q2 >

1
.
|qp|

De nuevo por el teorema 11 existe


43

p0
q0

tal que |q0 p0 | <

1
Q2

< |q p| (y si se quiere con (p0 , q0 ) = 1) de donde


mente.
(2) Si es racional, entonces =
p

| q | = |

aqpb
1
qb | qb

a
b

p
q

p0
q0

con a, b enteros. Para

y as sucesivap
q

, ba , tenemos

ya que |aq pb| 1 puesto que es un entero no nulo.

Ahora bien, si q > b entonces

1
bq

>

1
,
q2

luego q b lo que nos da una co-

ta superior para q y por lo tanto un nmero finito de


propriedad que buscamos.

p
q

que satisfacen la

Este resultado es optimal en el sentido siguiente:




p

Teorema 12. Si es algebraico, entonces  > 0 c > 0 tal que
q
p
c
.
q
q2+
Un mtodo para encontrar buenas aproximaciones racionales es el de las
fracciones continuas.

6.2.

Fracciones continuas

El principio de base de las fracciones continuas es el de establecer biyecciones entre los irracionales y las sucesiones de a0 , a1 , a2 , ... con ai
Z, ai > 0 si i , 0 y los racionales y las sucesiones finitas a0 , a1 , a2 , ..., an con
ai Z, ai > 0 si i , 0.
1
, >1y
1 1
1
luego pondremos a1 = [1 ]. Si a1 , 1 , escribiremos = a0 +
, 2 > 1
1
a1 +
2
y luego pondremos a2 = [2 ] y as sucesivamente.
Sea R, pongamos a0 = []. Si a0 , , escribiremos = a0 +

Si el algoritmo se termina en un nmero finito de etapas, entonces existe un n tal que an = n y tenemos
1

= a0 +
a1 +

1
a2 + .

44

.. + 1
an

Notacin:

a0 +
a1 +

= [a0 , , an ]

1
a2 + .

.. + 1
an

Si el algoritmo no se termina, obtenemos una sucesin a0 , a1 , a2 , con


a0 Z, ai N si i 1. Ms adelante mostraremos que en este caso es
irracional.
Ejemplos
(1)

187
= [5, 2, 1, 11]
35

(2) = [3, 7, 15, 1, 292, 1, 1, 2, ]


(3) = [1, 1, 1, ] donde es el nmero de oro, el cual satisface la ecuacin x2 x 1 = 0.
El siguiente teorema nos permitir justificar (por ejemplo) las escrituras
= [1, 1, 1, ], = [3, 7, 15, 1, 292, 1, 1, 2, ], etc.
Teorema 13. Si an , n n, la sucesin [a0 , a1 , ] converge hacia cuando


pn
pn
n . Adems si
= [a0 , a1 , , an ] con (pn , qn ) = 1, entonces
qn
qn
1
.
qn qn+1
Definicin 18. Los racionales

pn
se llaman los convergentes de .
qn

Corolario 5. Si es racional, el algoritmo de escritura en fraccin continua se


termina.
a
Demostracin. Si = , entonces por el teorema 13 tenemos que | bqn bpn |
b
| {z }
Z

p
b n
0 y por lo tanto debe existir un n tal que = n = [a0 , , an ]
qn
qn
Regresemos ahora el teorema 13:
Demostracin. Definimos por recurrencia las sucesiones siguientes: pn =
an pn1 + pn2 con p0 = a0 , p1 = a0 a1 + 1 y qn = an qn1 + qn2 con q0 = 1,
q1 = a1 .
45

Afirmacin 1: x > 0, [a0 , , an1 , x] =

xpn1 + pn2
si n 2.
xqn1 + qn2

xp1 + p0
,
xq1 + p0
supongamos ahora cierta la afirmacin 1 para n 2 y mostrmosla para
n+1

En efecto, procedamos por induccin sobre n 2: [a0 , a1 , x] =

[a0 , , an , x] = a0 +

1
= {y = an + } = [a0 , , an1 , y]
x

1
a1 + .

.. + a

n1 +

an +

1
x

luego por hiptesis de induccin


 xa +1 
n
pn1 + pn2
ypn1 + pn2
x

= xa +1 
[a0 , , an1 , y] =
=
n
yqn1 + qn2
qn1 + qn2
x

pn

z
}|
{
x (an pn1 + pn2 ) +pn1
=
x (an qn1 + qn2 ) +qn1
|
{z
}
qn

a p
+ pn2
luego poniendo x = an en la afirmacin se tiene [a0 , , an ] = n n1
=
an qn1 + qn2
pn
.
qn
Afirmacin 2: n 0, pn qn+1 pn+1 qn = (1)n+1
Como en la afirmacin 1, se procede por induccin sobre n.
Observemos que la afirmacin 1 nos dice que (pn , qn ) = 1 n, es decir
p
acabamos de mostrar que los n son los convergentes de .
qn


pn pn+1
1

=
La afirmacin 2 tambin implica que
.
qn qn+1
qn qn+1
p
p
Observacin: 2k , 2k+1 (se muestra tambin por induccin).
q2k
q2k+1


pn
1

La observacin implica en particular que
.
qn
qn qn+1

46

6.3.

Serie de ejercicios 8

1. Suponiendo que 3.1415926 es una aproximacin correcta de de


decimales, calcular los 4 primeros convergentes de .
2. Sea = [a0 , ..., an ] con a0 > 0. Mostrar que 1 = [0, a0 , ..., an ].

3. Determinar la fraccin continua de n2 + 1 para todo n 1.


4. Sean 1 y 2 dos races del polinomio x2 ax 1 con a N y 1 >
0. Mostrar que los denominadores de los convergentes de 1 estn
dados por
1n 2n
.
qn1 =
1 2
Deducir la frmula de n-simo nmero de Fibonacci.

6.4.

Teorema de Liouville

De nuestros primeros cursos de anlisis sabemos que los nmeros racionales son densos en los reales, esto quiere decir que todo nmero real
es el lmite de una sucesin de nmeros racionales. Hemos visto que una
forma eficaz de aproximar un nmero real por una sucesin de nmeros
racionales es la de utilizar fracciones continuas. Recordemos que el teorema de Dirichlet (teorema 11) nos dice que si R
existe una
\ Q entonces

p 1
p

infinidad de racionales con (p, q) = 1 tales que 2 . Como sabeq
q
q
mos, esto no es cierto si es racional. Tal vez sea ms interesante aproximar nmeros algebraicos. Desgraciadamente el siguiente teorema nos dice
que tales nmeros no se aproximan bien con nmeros racionales.
Teorema 14. (Liouville 1840) Sea R un nmero algebraico
de grado d


p c
mayor o igual a 2. Existe una constante c = c() tal que d para todo
q
q
p
racional .
q
p

Demostracin. Sean q un nmero racional cualquiera, P polinomio minip


mal de (entonces deg P = d). Notemos que P ( q ) , 0, si no dividimos P
p
por (X q ) y eliminamos los denominadores del polinomio resultante para
obtener Q Z[X] con Q() = 0 y deg Q = deg P 1, lo cual no es posible
ya que P es el polinomio minimal de .

47


 p 
p
Tenemos que 0 , qd P ( q ) Z y entonces qd P q 1 luego

!
p
d
P
q
q

(4)

Por otra parte, el teorema de los valores intermediarios se tiene


!
!
p 0
p
p() p
= P ()
q
q
p

para un cierto [, q ] o [ q , ]. Pongamos c()1 :=

max

1x+1

|P 0 (x)|

(observe que c() , 0 ya que al menos P 0 () , 0). Finalmente la ecuacin


(4) y el hecho que P () = 0 nos dan


p c()

q
qd

Una generalizacin de este resultado es el teorema de Roth (que no demostraremos ac):


Sea un numero
irracional algebraico,  > 0 c = c(, ) tal que p Z

p
c
q N 2+ .
q
q
El teorema de Roth muestra que la medida de irracionabilidad de un nmero algebraico es 2, ste teorema y el de Liouville permiten demostrar,
por contraposicin, la irracionalidad de muchos nmeros como veremos a
continuacin.

6.5.

Nmeros trascendentales

En esta seccin mostraremos que existe una infinidad de nmeros trascendentales.


Proposicin 13. Sea b > 1 entero y (an )n1 una sucesin de enteros comprendidos entre 0 y b 1, el nmero real

X
an
O :=
bn!
n=1

es trascendente.
48

Demostracin. El criterio del cociente nos garantiza la convergencia de O.


n
X
ak
n!
Pongamos qn = b y pn = qn
, entonces
bk!
k=1

X
X
b1
1
ak
b1
1
O pn =

<
= (n+1)!1 n


k!
k!
k
qn
qn
b
b
b
b
k=n+1
k=n+1
k=(n+1)!
Ahora si O fuese algebraico de grado d, por el teorema de Liouville (teop
rema 14) debe existir una constante c tal que para todo racional q , O se
tenga


O p c

q qd
Pero entonces


pn 1
c

O < n
qn
qn
qnd
lo cual implica que c qndn para todo n, es decir la relacin slo es posible
si c = 0, lo que contradice el teorema 14 ya que c > 0.
Cantor demostr en 1874 que R es no numerable y que el conjunto de
nmeros algebraicos Q es numerable. Con la proposicin precedente acabamos de construir una infinidad de nmeros trascendentales non numerables, exactamente 20 , llamados trascendentes de Liouville. Desde luego,
no son los nicos:

X
1
es trascendente.
Teorema 15. (Hermite 1873) El nmero e :=
n!
n=1
Zt
Demostracin. Etapa 1: Sea I(t) =
etx f (x)dx donde f es un polinomio
0

a coeficientes reales de grado deg f = m. Integrando por partes


Zt
h
it Z t
tx
tx 0
t
I(t) = e f (x)
e f (x)dx = f (t) + e f (0) +
etx f 0 (x)dx =
0

= iterar = et

m
X

f (j) (0)

j=0

m
X

f (j) (t)

j=0

Etapa 2:
Z t
Z


tx
e |f (x)| dx =
|I(t)|
0

t
0

etx (|a | |x|m + + |a0 |)dx tet f(t)


|{z} m |{z}
et

49

t m

donde f(x) = |am |xm + + |a0 |


Etapa 3: Supongamos que e es algebraico de grado n, existe entonces una
relacin a0 + a1 e + + an en = 0 con a0 , 0, aj Z.
Sea f (x) = xp1 (x 1)p (x 2)p (x n)p con p primo grande. El grado de f
es m = deg f = pn + p 1. Podemos escribir f (x) = g(x)(x k)p k = 1, 2, , n.
Derivando j veces tenemos
f

(j)

!
j
X
di
j (ji)
=
g
(x) i (x k)p
i
dx
i=0

de donde f

(j) (k) = 0

si j < p y f

(j)

!
j (jp
g
(k)p! si j p.
(k) =
p

Etapa 4: Sea J = a0 I(0) + a1 I(1) + + an I(n), por la etapa 1 se tiene

m
m
m
m
X

X
X

J = a0 e0
f (j) (0)
f (j) (0) + + an en
f (j) (0)
f (j) (n)

j=0

j=0

luego por la etapa 2


J =

j=0

m X
n
X

j=0

ak f (j) (k)

j=0 k=0

Etapa 5: Para todo j, f (j) (k) es un entero divisible por p! con k = 1, ! , n.


j
Para j < p 1 tenemos f (j) (0) = 0 y si j p 1 se tiene f (j) (0) =
(p
p1
f (x)
1)!h(jp+1) (0) donde h(x) = p1 . Sigue entonces que (p 1)! | f (p1) (0) pero
x
p - f (p1) (0)
Etapa 6: Elijamos ahora p primo tal que p > a0 , , an de forma que p a1 , , a2 .
J = a0 f

(p1)

(0) a0

m
X

j=p

(j)

(0)

m X
n
X

ak f (j) (k)

i=p k=1

Ahora bien, p divide todos los trminos de la expresin de arriba excepto


el primero, sigue entonces que J , 0. Como (p 1)! divide todos los trminos de J no nulo, se tiene que |J| (p 1)!

50

Etapa 7: Para f (x) = xp1 (x 1)p (x n)p , si k n se obtiene


f(k) (2n)m
donde m = np + p 1 2np. Luego
|J|

etapa 2

|a1 |ef(1) + + |an |nen f(n)

n
X

|ak |kek (2n)2np

k=0

| {z }
:=D(n)

Etapa 8: Finalmente hemos mostrado que


(p 1)! |J| D(n)(2n)2np
luego enviando p llegamos a una contradiccin ya que p! crece ms
rpido que cp .

6.6.

Serie de ejercicios 9

1. Determinar el nmero racional cuya fraccin continua es [1, 2, 3, 4].


2. Mostrar que, para todo irracional, los denominadores
de los con
(1+ 5)n1
. Adems, mos 2
n
2
A+ A +4
qn
.
2

vergentes de satisfacen la desigualdad qn


trar que si an A para todo n, entonces

3. Sea R. Supongamos que en la fraccin continua que representa a


2
, la sucesin an sea creciente. Mostrar que si an+1 > (an + 1)n para
todo n, entonces es trascendente.


pn

Indicacin: Utilizar el hecho que qn < 1 2 .
an+1 qn

4. Admitamos el teorema de Roth. Mostrar que la suma ab +ab +ab +


es trascendente para todos los enteros a 2, b 3.
Teorema de Roth: Para todo irracional algebraico y todo  > 0, existe
p
una constante c = c(, ) tal que para todo racional q , se tiene


p > c .

q q2+
51

A continuacin presentamos un teorema (sin demostracin) que nos dar


un nuevo criterio de trascendencia.
Teorema 16. (Lindemann 1882) Sean 0 , , n nmeros algebraicos distintos y 0 , , n nmeros algebraicos no todos nulos. Entonces 0 e0 + +
n en , 0.
Corolario 6. Los nmeros siguientes son trascendentes: e , , log , sin ,
cos , tan con algebraico , 0, 1.
No vamos a probar todo el corolario, sin embargo estudiaremos los casos e y :
Supongamos e algebraico, entonces existe una relacin 0 zn + 1 zn1 +
+ n = 0 con i Z no todos nulos y z = e . De nuestros cursos de lgebra sabemos que Q() es un cuerpo, en particular para k = 1, , n tenemos
que k = k es algebraico, lo que contradice el teorema 16.
Para ver que es trascendente (un resultado muy importante que combinado con la teora de Galois puso fin al problema de la cuadratura del
crculo) observemos que (1) e0 + 1 e2i = 0 y de nuevo si suponemos
algebraico se contradice el teorema 16.
Recordemos que el teorema de Roth nos dice que si es un nmero algebraico no racional, entonces para todo
 > 0
existe una constante c = c()
p
p
c
tal que para todo racional se tiene 2+ . Este teorema nos perq
q
q
mitir mostrar el siguiente resultado concerniente a las soluciones enteras
de ciertos tipos de ecuaciones polinomiales homogneas en dos variables.
Teorema 17. (Thue 1909) Sea F(x, y) = a0 xn + a1 xn1 y + + an y n una forma
irreducible de grado n 3, ai Z y sea m Z. La ecuacin F(x, y) = m tiene a
lo sumo un nmero finito de soluciones (x, y) Z2 .
Demostracin. Por irreductibilidad de F podemos escribir
a0 (x 1 y) (x n y) = m
con todos los i algebraicos no racionales diferentes.
Supongamos por el absurdo que la ecuacin F(x, y) = m posee una infinidad de soluciones, por el rol simtrico que tienen x e y podemos suponer
sin prdida de generalidad de hay una infinidad de y tales que y .
52

Pongamos  := min d(i , j ) y dividamos la expresin de arriba por y (que


i,j

podemos suponer non nulo), tenemos


!
!
x
x
m
1 n =
y
y
a0 y n
Como y puede ser elegido arbitrariamente grande, uno de los factores
a la
x


izquierda entre parntesis debe ser menor a , digamos que 1 .
2
y
2


x

Debido a la eleccin de , para el resto se tiene i > si i , 1, de
y
2
donde


x C
1
y
|y|n
donde C = C(, m, a0 ). Pero segn el teorema de Roth, esto no es posible.
En particular este teorema nos dice que x3 2y 3 = 11 posee un nmero
finito de soluciones, algo que no es trivial de ver si uno no conoce este
teorema. La hiptesis sobre el grado de la forma es importante, he aqu un
contra-ejemplo:
Definicin 19. Sea d un entero positivo sin factores cuadrados. Las ecuaciones
de la forma x2 dy 2 = 0 se llaman ecuaciones de Pell.
Ms adelante veremos una demostracin sobre la infinitud de soluciones de una ecuacin de Pell.
Un fenmeno similar al teorema de Thu ocurre con las ecuaciones de tipo
xn + y n = zn de las cuales buscamos todas las soluciones enteras. Sabemos
que para n = 2 las soluciones a esta ecuacin se conocan desde hace tiempo, son la llamadas ternas pitagricas: x = m2 n2 , y = 2mn, z = m2 + n2 para
todos m, n Z. Por el contrario, el caso n 3 conocido como ltimo teorema de Fermat no posee ms que la solucin entera trivial, este teorema fue
demostrado en 1994 por Andrew Willes.
Proposicin 14. Sea x una solucin de xm + c1 xm1 + + cm = 0 ci Z,
entonces o bien x Z o bien x es irracional.
Demostracin. Supongamos que x = ba con (a, b) = 1, tenemos que am +
c1 am1 b + + cm bm = 0 b | am , pero (a, b) = 1, luego b = 1 y entonces
x es un entero.
53

Por ejemplo esta proposicin


generaliza la posiblemente famosa demos
tracin del hecho que 2 es irracional.
Sabemos por el teorema 16 que e es trascendente y por lo tanto irracional. Sin embargo, no necesitamos de invocar al teorema de Lindemann
para probar la irracionalidad de e
Proposicin 15. e es irracional.
Demostracin. Supongamos que e = ba . Para k > b tenemos que b | k! y entonces


1
1 1
Z
:= k! e 1
1! 2!
k!
Pero
0<=

1
1
1
1
1
+
+
+
+ = < 1
2
k + 1 (k + 1)(k + 2)
(k + 1) (k + 1)
k

lo cual no es posible ya que es un entero y por lo tanto no puede estar


estrictamente comprendido entre 0 y 1.
A continuacin enunciaremos otro teorema de Lindemann sobre la
trascendencia de , la cual no demostraremos en este curso.
Teorema 18. (Lindemann 1877) es trascendente.
Demostracin. Omitida.
Corolario 7. La cuadratura del crculo es imposible.
Demostracin. Ver curso de lgebra de segundo ao.
Para no irnos con las manos vacas, vamos a demostrar que es irracional.
Este hecho fue probado por Lambert en 1750. La prueba que presentaremos es ms corta y no corresponde a la original.
Teorema 19. es irracional.
2n

xn (1 x)n
1 X
Demostracin. Sea fn (x) =
=
cm xm cm Z. Para 0 < x < 1
n!
n! m=n
se tiene que 0 < fn (x) <

1
n! .

Observemos tambin que f (x) = f (1 x) luego

0
m<n

(m)
m!
m (m)
fn (1) = (1) fn (0) =
n! cm n m 2n

0
m > 2n
54

(m)

(m)

en decir fn (0), fn (1) Z. Supongamos que 2 = ba , sea




G(x) = bn 2n f (x) 2n2 f (2) (x) + + (1)n f (2n) (x)
entonces para f de la forma de cualquier fj (x) se ve de inmediato que
G(0), G(1) Z.
Por otra parte
d
(G0 (x) sin x G(x) cos x) = (G00 (x) + 2 G(x)) sin x =
dx
= bn (2n f 00 (x) + + (1)2 f 2n+2 (x) + 2n+2 f (x) + + (1)2 f (2n) (x)) sin x
= bn (2n+2 f (x)) sin x = 2 an f (x) sin x
entonces
Z1
1
an sin xf (x)dx = [G0 (x) sin x G(x) cos x]10 = G(0) + G(1) Z

0
pero
Z

0<
0

an
an sin x f (x) <
<1
|{z} |{z}
n!
(0,1) 0<f (x)< 1
n!

si n es suficientemente grande, lo cual no es posible.

55

7.
7.1.

Cuerpos cuadrticos
Cuerpo de nmeros

Un cuerpo de nmeros K es una extensin finita de Q. En esta seccin


abordaremos de una manera simplificada ciertos aspectos de la teora de
cuerpos estudiada en cursos de lgebra de segundo ao. Recordemos que
un nmero C es algebraico si existe p Q[X] tal que p() = 0. El con
junto de nmeros algebraicos es denotado por Q.
El polinomio minimal P de es el polinomio de grado minimal que se
anula en y tiene como coeficiente principal (coeficiente del trmino de
ms alto grado) igual a 1. Como las otras races de P comparten el mismo polinomio minimal que , a menudo se las llama conjugadas de y
podemos notarlas por 1 = , , d donde d = deg P .
Definicin 20. Sean un nmero algebraico y 1 , , d las conjugadas de
su polinomio minimal, llamamos norma de a la expresin:
N () = 1 d
y Traza de a:
T () = 1 + + d
Observemos que si P (X) = X d + a1 X d1 + + ad es el polinomio minimal de , tanto N () como T () son racionales ya que N () = (1)d ad y
T () = a1 .
Definimos Q() := {p() | p Q[X]} = {b0 + b1 + + bd1 d1 , bi Q}.
Observaciones:
1. La segunda igualdad viene del algoritmo de la divisin sobre el anillo
de polinomios Q[X]: p Q[X], p(X) = a(X)P (X) + r(X) con deg r
d 1 y P polinomio minimal de , luego evaluando en se tiene
p() = r() = b0 + + bd1 d1 .
2. En un contexto ms general, el conjunto Q() se define como el ms
pequeo cuerpo intermediario entre Q y una extensin de Q conteniendo .
De nuestra definicin, vemos inmediatamente que Q() es un Q-espacio
vectorial de dimensin d con base {1, , 2 , ..., d1 } (para ver que es una
56

familia Q-linealmente independiente utilizar el hecho que el grado del


polinomio minimal de es d). Decimos entonces que el grado de Q() sobre Q es d y escribimos [Q() : Q] = d.
Nuestra definicin nos dice tambin que Q() es un anillo, pero no que se
trata de un cuerpo. Sean entonces 0 , q Q() y P el polinomio minimal
de con deg P = d, tenemos que q y P son relativamente primos (como polinomios en X) ya que deg q d 1 y P es irreductible (si no P no sera minimal), entonces por el algoritmo de la divisin euclidiana existen polinomios a, b Q[X] con deg a, deg b d 1 tales que a(X)q(X) + b(X)P (X) = 1,
luego evaluando en se tiene a()q() = 1. Q() es entonces un cuerpo.
Ejemplos

(1) = i = 1
Q(i) se llama cuerpo Gaussiano. El polinomio minimal de i es P (X) =
X 2 + 1, las conjugadas de i son entonces 1 = = i y 2 = i. Luego
N (i) = i (i) = 1, T (i) = i + (i) = 0.

(2) = 2

El polinomio
minimal
de
2 es P(X) = X 2 2,

sus conjugadas son


= 1 = 2 y 2 = 2, luego N ( 2) = 2, T ( 2) = 0.
Definicin 21. Un nmero algebraico se llama entero algebraico si su polinomio minimal P Z[X].
Observemos que por definicin el polinomio minimal P de un entero algebraico es unitario, es decir el coeficiente del trmino de ms alto grado es
igual a 1. Por otro lado, si es un entero algebraico sigue inmediatamente
que N (), T () Z (simplemente comparar con los coeficientes de P ).
Es posible mostrar que los enteros algebraicos forman un anillo, a menudo denotado por O. Para K Q una extensin finita de Q (es decir un
cuerpo de nmeros), definimos por OK := O K el conjunto de los enteros
algebraicos de K. Este conjunto es tambin un anillo llamado anillo de los
enteros algebraicos de K.
Definicin 22. Un entero algebraico (sobre K) , 0 es una unidad si

1
OK .

Ejemplo Si tomamos K = Q, entonces OQ = Z, es decir los enteros algebraicos de Q son los enteros que conocemos. En efecto, si OQ existe
una relacin de tipo d + a1 d1 + + ad = 0 con los ai Z, luego por la
57

proposicin 14 o bien Z o bien es racional, en el segundo caso el polinomio minimal de es un monomio unitario, entonces tiene que ser
entero. Tenemos entonces que las unidades de OQ son {1, 1}.
Definicin 23. Sean , , OK
(1) Decimos que es divisible por 0 , si

OK .

(2) Decimos que y estn asociados si existe OK unidad tal que = .


(3) Decimos que es irreducible si para todo tal que | , entonces est
asociado a o bien es una unidad.
(4) Decimos que es primo si para todos y tales que | , entonces
| o bien | .

7.2.

Cuerpos cuadrticos

Sea 0, 1 , d Z sin factores cuadrados, un cuerpo cuadrtico es una


extensin algebraica
de Q de grado 2. Se puede mostrar fcilmente que la
asociacin d 7 Q( d) nos da una biyeccin entre los cuerpos cuadrticos
y tales d. Entonces de forma ms explicita un cuerpo cuadrtico es un
cuerpo de la forma

Q( d) = {u + v d | u, v Q}
con d , 0, 1 sin factores cuadrados.

A continuacin vamos a determinar los enteros algebraicos de Q( d)

Proposicin 16. dado un cuerpo de nmeros K = Q( d) con d , 0, 1 sin


factores cuadrados, el anillo de enteros algebraicos de K es un anillo de la forma
Z[] = {m + n | m, n Z} donde

d
si d 2, 3 mod 4

=
1+ d

si d 1 mod 4

Demostracin. Sea = u + v d un entero algebraico. Pongamos a = 2u y


2
b = 2v. El
polinomio minimal de es P (X) = X aX + c, en efecto sea
Como
= u v d el conjugado de , tenemos que + = 2u = a y c = .
es un entero sobre K, se sigue que a, c Z.
58

4c = 4 = 4N () = 4(u + v d)(u v d) = a2 db2 a2 4c = db2 Z,


m2
ahora si b = m
n tendramos que n2 d Z, pero d es sin factores cuadrados,
luego n = 1 de donde b Z.
Tenemos entonces que a, b y c son enteros, nos resta estudiar las posibilidades para d:
Caso 1: Si d 2, 3 mod 4
mod 4

4c = a2 db2 = a2 2b2 mod 4 o a2 3b2 mod 4. Ahora bien, como


x2 0, 1 mod 4 para todo x Z, las dos ecuaciones modulares de arriba no
son posibles a menos que a2 b2 0 mod 4, luego a y bdeben ser pares y
entonces u, v Z. Se tiene entonces que OQ(d) = {m + n d | m, n Z}.
Caso 2: Si d 1 mod 4
mod 4

4c = a2 db2 = a2 b2 mod 4 a b mod 2, es decir a y b son ambos


pares o impares, en todo caso ab
2 Z, de donde u v Z. Luego podemos
escribir

1+ d
u + v d = (u v) + v(1 + d) = u v +b
|{z}
2
Z

sobre
de este modo los enteros algebraicossobre Q( d) son engendrados


Z por {1, 1+2 d }. Finalmente OQ(d) = m + n 1+2 d | m, n Z


Ejemplos
(1) 1 3(4), luego por la proposicin
16 tenemos que el anillo de enteros Gaussianos es Z[i] = {m + n d | m, n Z}.
 

(2) 5 1(4), luego el anillo de enteros algebraicos de Q( 5) es Z 1+2 5 =





1+ 5
m + n 2 | m, n Z .

7.3.

Unidades

En el contexto de un cuerpo cuadrtico, es fcil verificar que la norma

d, su
de un nmero algebraico

es
multiplicativa.
En
efecto,
si

=
u
+
v

conjugado es = u v d, luego para otro nmero algebraico un clculo


elemental da N () = N ()N ().
59

Proposicin 17. Sea un entero algebraico, es una unidad si y slo si N () =


1.
Demostracin. ) Si es una unidad, por definicin 1 es algebraico, luego
1 = 1 N ()N ( 1 ) = N (1) = 1, pero N (), N ( 1 ) Z, luego necesariamente N () = 1.
) Sea un entero algebraico tal que N () = = 1 la afirmacin sigue
trivialmente.

Para un cuerpo cuadrtico Q( d) es fcil demostrar que si d < 0, y


d 2, 3(4) el nmero de unidades es finito. En efecto hay que estudiar
los valores enteros de la ecuacin x2 dy 2 = 1, descartamos la ecuacin
x2 dy 2 = 1 ya que su nica solucin entera es (x, y) = (0, 0), luego si consideramos la ecuacin x2 dy 2 = 1 es claro que esta ecuacin no puede
tener ms que un nmero finito de soluciones enteras. Si d > 0 el nmero
de unidades es infinito, ste hecho es una consecuencia directa del teorema
de Dirichlet de las unidades (que no demostraremos ac ya que corresponde
a una rama de la teora de nmeros que no est al alcance de este curso
introductorio: la teora algebraica de nmeros).
Antes de terminar esta seccin examinemos un ejemplo interesante:

Consideremos el cuerpo cuadrtico K = Q( 5). En el anillo de enteros


de este cuerpo se tiene la factorizacin

6 = 2 3 = (1 + 5)(1 5).
Vamos a mostrar que la factorizacin de 6 en producto de irreducibles no
es nica (nica en el sentido mdulo una unidad). En efecto, la escritura de arriba no
es suficiente para concluir, ya que a priori los factores
2, 3, 1 + 5, 1 5 podran estar asociados. Como 5 < 0 y 5 3(4) tenemos que las unidades de OK son {1, 1}.

Mostraremos que 2 es irreducible (para 3 y 1 5 el razonamiento es


similar). Si 2 = entonces 4 = N () N () luego los valores posibles para
|{z} |{z}
Z

N () son 4, 2, 1, pero como N () = x2 + 5y 2 los valores 2 no pueden


ser posibles, en consecuencia es una unidad o bien es una unidad.
Podemos aprovechar
tambin
para mostrar que 2 no es primo. En efec

to 2 | 6 = (1 + 5)(1 5), entonces si 2 fuese primo el debe dividir


60

(1 + 5) o(1 5), luego debe existir Z[ 5] tal que 1 5 = 2


6 = N (1 5) = N (2)N () = 4 N () lo cual no es posible.
|{z}
Z

Recordemos que una anillo es factorial si todos sus elementos pueden


escribirse de manera esencialmente nica como producto
de factores irreducibles. Luego el anillo de enteros algebraicos de Q( 5) no es factorial.
Proposicin 18. OK es un anillo factorial si y slo si todo elemento irreducible
de OK es tambin primo.
Recordemos que en un anillo ntegro, todo elemento primo es irreducible. El anillo
de enteros algebraicos de un cuerpo de nmeros cuadrticos

K = Q( d) es un anillo factorial si y slo si d = 1, 2, 3, 7, 11, 15,


43, 67, 163. Este resultado es ms difcil de establecer (con teora algebraica de nmeros se puede mostrar gracias a la finitud del nmero de
clases de ideales hK que para tales d, OK es principal y por lo tanto factorial). Si d > 0 no se conoce muy bien lo que pasa, de hecho no se sabe si
hay o no una infinitud de anillos factoriales.

7.4.

Serie de ejercicios 10

1. Encontrar una infinidad


diferentes en los anillos de en
de unidades
teros algebraicos de Q( 2) y Q( 3).

2. Mostrar que la ecuacin
2 3 = 6 6 implica que el anillo de
enteros algebraicos de Q( 6) no es factorial.

3. Explicar por qu la ecuacin 2 11 = (5 + 3)(5 3) no constituye


una contradiccin
con el hecho que el anillo de enteros algebraicos

de Q( 3) es factorial.

4. Mostrar que el anillo de enteros algebraicos de Q( 10) no es factorial.

Indicacin: Considerar el entero algebraico 4 + 10.

7.5.

El cuerpo Gaussiano Q(i)

Definicin 24. Un cuerpo cuadrtico K = Q( d) es euclidiano si su anillo


de enteros algebraicos OK posee una divisin euclidiana, es decir
, OK , , 0 , OK tal que = + , |N ()| < |N ()|
61

La existencia de la divisin euclidiana en un anillo implica la existencia


de un mximo comn divisor (mdulo una unidad), adems de una relacin
de Bzout:
, OK , , OK tal que + = (, ) := d
Adems si d es una unidad, podemos elegir 0 , 0 OK tales que 0 +0 =
1.
Proposicin 19. Si OK es euclidiano, entonces tambin es factorial.
Demostracin. Sea irreducible. Supongamos que | y que - , como
es irreducible son nicos divisores son , donde es una unidad.
Deducimos que = (, ) = 0 + 0 (Bzout), de donde dividiendo por
se tiene 1 = + , luego = + es decir | .
Teorema 20. K = Q(i) es euclidiano.
Demostracin. Sean , OK , , 0. Como es invertible en Q(i) se tiene = u + iv con u, v Q. Sean x e y los enteros ms prximos a u y v
respectivamente. Pongamos r = u x, s = v y, = x + iy y = (r + is).
Tenemos que |r|, |s| 12 y = + con = OK , adems N () =
N ()N (r + is) = N ()(r 2 + s2 ) < N () ya que r 2 + s2 14 + 14 < 1.
Resumiendo un poco los resultados que hemos obtenido:
1. Q(i) es euclidiano y por lo tanto factorial, en particular un elemento
de Q(i) es primo si y slo si es irreducible.
2. OQ(i) = Z[i].
3. Las unidades de Z[i] son {1, i}.
Lema 10. Sea OQ(i) . Si N () {2, 3, 5, 7, 11, 13, ...} (los primos), entonces
es un primo gaussiano.
Demostracin. Supongamos que = , entonces N () = N ()N ()
N () = 1 o bien N () = 1
Lema 11. Sea OQ(i) un primo gaussiano. Existe un nico nmero primo p
tal que | p en OQ(i)

62

Demostracin. Observemos que | = N () Z. Elijamos el ms pequeo de los enteros positivos divisibles por , digamos p. Un tal p debe
ser necesariamente primo ya que si p = mn tendramos que | mn = p,
pero es un primo gaussiano y entonces | m o bien | n lo que contradice la minimalidad de p, luego m = p o n = p de donde sigue que p es primo.
Para la unicidad, observemos que si existe otro primo p0 divisible por , la
relacin de Bzout en Z nos dice que existen a0 , a0 Z tales que ap+a0 p0 = 1,
luego | ap + a0 p0 = 1 entonces debe ser una unidad lo que no es posible
ya que es primo, sigue entonces que p = p0 .
En lo que resta de esta seccin K = Q(i).
Lema 12. Sea p un nmero primo, p es un primo gaussiano o bien el producto

de dos primos gaussianos conjugados p = .


Demostracin. Sea p nmero primo y OK tal que | p, es decir p =
OK . Tenemos que p2 = N (p) = N ()N (), luego si N () = 1 entonces
es una unidad y por consiguiente p es un primo gaussiano. Si N () =
p, por el lema 10 debe ser un primo gaussiano (del mismo modo ) y
necesariamente = ya que Z.
Es natural preguntarse en qu caso nos encontramos dado un nmero
primo p. La respuesta est en el siguiente teorema.
Teorema 21. Los primos gaussiano son todos los nmeros primos p 3 mod 4,
los factores primos de p 1 mod 4 en p = y por 1 + i.
Antes de comenzar la prueba de este teorema observemos que esta clasificacin es "mdulo una unidad", por ejemplo 1i es tambin irreducible
(de manera equivalente primo) pero 1 i = i(1 + i). Ahora bien, gracias a
la proposicin 17 podemos ver que las nicas unidades de OK son {1, i}.
Demostracin. Si p 3 mod 4 N () , p ya que N () = x2 +y 2 0, 1, 2 mod 4.
Si p 1 mod 4 la ley de reciprocidad cuadrtica nos dice que 1 es un
resto cuadrtico mdulo p, luego p | x2 + 1 = (x i)(x + i) para un cierto
x Z. Si p fuese un primo gaussiano entonces p | x i o bien p | x + i, en
ambos casos se tiene px pi < OK .
Se muestra fcilmente que 1 + i debe ser
Si p = 2, 2 = (1 + i)(1 i) = .
primo gaussiano.

63

7.6.

Ecuaciones de Pell

Sea d Z sin factores cuadrados positivo, nos interesamos en las soluciones enteras de la ecuacin
x2 dy 2 = 1

(5)

fcilmente vemos una de tipo (x, y) = (1, 0). Supongamos ahora que (x, y)
es una solucin entera positiva de la ecuacin (5), es decir con x, y > 0,
entonces

1
1
xy d =
> 0 x > y d xy d <
x+y d
2y d
de donde se deduce



d x < 1

y 2y 2

Vamos a admitir dos resultados sobre las fracciones continuas (para ms


detalles consultar: A Comprehensive course in Number Theory de Alan Baker).

p
p
1

1. Si < 2 entonces es un convergente de = d.
q
q
2y

2. La fraccin continua de d es de la forma [a0 , a1 , , am ]. El nmero


m se llama periodo.

p
Notemos por n los convergentes de d y n los cocientes completos de
qn

d. Por la observacin precedente existe n N tal que x = pn , y = qn ,


es decir pn2 dqn2 = 1. La teora de fracciones continuas estudiadas en el
captulo precedente nos dice que

d=

pn n+1 + pn1
qn n+1 + qn1

(6)

de donde, utilizando pn+1 qn pn qn1 = (1)n , tenemos

qn d pn =

(1)n
qn n+1 + qn1

p
Ahora si n es par tenemos qn d pn > 0, pero 2k = d entonces n
q2k
debe ser impar.
64

De hecho n debe ser un impar de


la forma lm 1 con l un entero, m el
periodo de la fraccin continua de d. De manera equivalente l N si m
est par, l 2N si m es impar.

En efecto, de la ecuacin (6) tenemos (pn qn d)n+1 = qn1 d pn1 de


donde

(pn2 dqn2 )n+1 = (qn1 d pn1 )(pn + qn d) = (1)2 d + c, c Z

como n es impar y pn2 dqn2 = 1 sigue que n+1 = d + c. De otra parte

1
d = a0 +
con 1 = m+1 = 2m+1 , etc.
1

1
1
1
Adems n+1 = an+1 +
an+1 = n+1
= d+c
= a0 +
n+2
n+2
n+2
1
1
1
1
c+

Z. Ahora bien como 0 <


,
< 1, necesariamente
1 n+2
1 n+2
1
1

= 0 1 = n+2 y como el periodo es m entonces n = lm 1


1 n+2
(con n impar).
Veamos ahora que esta condicin necesaria sobre n es tambin suficiente.
Teorema 22. Todos los convergentes descritos anteriormente, es decir con n
impar de la forma ml 1 son las soluciones de la ecuacin (5).
Demostracin. Para todo n = lm 1 impar se tiene 1 = n+2 y entonces

p
+ pn
p + pn
1
1
d = n+1 n+2
= n+1 1
. Pero d = a0 +
1 =

qn+1 n+2 + qn
qn+1 1 + qn
1
d

a
0

pn+1 a0 pn + pn d
d=
y entonces qn d + (qn+1 a0 qn ) d = (pn+1 a0 pn ) +
qn+1 a0 qn + qn d
pn d, como d es irracional (sin factores cuadrados) sigue que
qn d = pn+1 a0 pn , pn = qn+1 a0 qn
finalmente a0 =
impar).

pn+1 qn d qn+1 pn
=
pn2 dqn2 = pn qn+1 pn+1 qn = 1 (n
pn
qn

De manera similar se demuestra:

65

Teorema 23. La ecuacin x2 dy 2 = 1 (d sin factores cuadrados) no tiene


soluciones si el periodo m de d es par. Si m es impar, x = pn , y = qn con

pn
convergente de d, n = ml 1 (l N ) representan todas las soluciones
qn
positivas.

7.7.

Serie de ejercicios 11

1. Mostrar que el anillo de enteros algebraicos de Q( 3) es euclidiano.


2. Utilizar el ejercicio precedente
para mostrar
que existen
enteros al
gebraicos y en Q( 3) tales que (1 + 2 3) + (5 + 4 3) = 1.
3. Encontrar la solucin positiva minimal de la ecuacin x2 39y 2 = 1.
4. Determinar si la ecuacin x2 31y 2 = 1 admite una solucin utilizando la teora de las ecuaciones de Pell.

66

8.

La conjetura ABC

Los conjuntos Z y C[t] tienen muchas cosas en comn, como anillos podemos efectuar ciertas operaciones entre sus elementos como la adicin, la
multiplicacin. Adems ambos son euclidianos y por lo tanto factoriales,
lo que nos permite escribir para todo elemento n Z:
n=

r
Y

mi

pi

i=1

con {1} y los pi primos (=irreductibles). Anlogamente, para todo


polinomio p(t) C[t]:
s
Y
(t i )mi
p(t) = c
i=1

ya que C es algebraicamente cerrado, los polinomios irreductibles son los


monomios t . Observemos que los i son las races de p(t) con multiplicidad mi correspondiente.
Esta analoga entre Z y C[t] es sin embargo ms profunda.

8.1.

ABC para los polinomios

Q
Sea f C[t], f , 0. Escribamos f (t) = c ri=1 (ti )mi con los i distintos.
Sean
X
deg f :=
mi
i=1

y
n0 (f ) := r.
Observemos que deg f puede ser grande an si n0 (f ) es pequeo, por
ejemplo si f (t) = (t )1000 , deg f = 1000, n0 (f ) = 1.
Teorema 24. Sean f , g, h C[t] polinomios no constantes relativamente primos tales que f + g = h. Entonces
max(deg f , deg g, deg h) n0 (f gh) 1
Antes de dar la demostracin de este teorema, veamos un resultado
particularmente interesante que se deduce de l:

67

Corolario 8. (Fermat para los polinomios) Si n 3 no hay ninguna solucin a la ecuacin:


x(t)n + y(t)n = z(t)n
donde x, y, z C[t] polinomios no constantes relativamente primos.
Demostracin. Sean f (t) = x(t)n , g(t) = y(t)n y h(t) = z(t)n . Por hiptesis
f + g = h, luego por el teorema ABC para los polinomios (teorema 24)
tenemos
max (deg xn , deg y n , deg zn ) n0 (xn y n zn ) 1 = n0 (xn ) + n0 (y n ) + n0 (zn ) 1
ya que los polinomios x, y, z son relativamente primos, es decir sin races
comunes. Ademas las relaciones deg xn = n deg x, n0 (xn ) = n0 (x) deg x
nos dicen que
max (n deg x, n deg y, n deg z) deg x + deg y + deg z 1
de donde
n(deg x + deg y + deg z) 3(deg x + deg y + deg z) 3
y luego
(n 3) (deg x + deg y + deg z) 3
|
{z
}
>0

entonces necesariamente n < 3.


Falsos ejemplos

(1) t 3 + ( 3 2t)3 = ( 3 3t)3 . Este ejemplo no contradice el corolario 8 ya que


si dividimospor t 3 para
tener polinomios relativamente primeros tenemos 13 +( 3 2)2 = ( 3 3)2 , ahora bien estos polinomios son constantes
y el corolario no se aplica.
(2) (t a)2 + (2ta)2 = (t 2 + a2 )2 . Este ejemplo nos muestra que como en el
caso de los nmeros, Fermat es cierto para n 2.
(3) (x + 1)p = xp + 1p (mod p). Es decir este resultado no se aplica para
todos los cuerpos.

68

8.2.

Preparativos para la prueba del teorema ABC para los


polinomios

Definicin 25. Sea f una funcin compleja non nula, llamamos derivada
logartmica de f a la funcin
L(f ) :=

f0
f

En los puntos en los que f no se anula, la derivada logartmica coincide con la


composicin de la funcin |f | y la funcin log, es decir log |f | como lo muestra
la regla de la derivacin en cadena.
Las siguientes propiedades de la derivada logartmica son un ejercicio
elemental de verificacin:
1. L(f1 fn ) =

n
X

L(fi )

i=1

!
f
2. L
= L(f ) L(g)
g
r
r
Y
X
mi
mi
3. Si f (t) = c
(t i ) entonces L(f )(t) =
t i
i=1

i=1

Comencemos ahora la prueba del teorema 24


Demostracin. Tenemos f + g = h

g
f
+
= 1 R0 + S 0 = 0
h
h
|{z} |{z}
R

L(R)
R0 /R
S
S
= , o si se prefiere
= .
0
S /S
R
L(S)
R
g g/h S
g
L(R) L(f ) L(h)
Luego =
= =
=
si utilizamos la propiedad
f
f /h R
f
L(S) L(g) L(h)
2 de la derivada logartmica.
Q
Q
Q
Si f (t) = cf (t i )mi , g(t) = cg (t j )nj y h(t) = ch (t k )lk . Segn
la propiedad 3:
P mi P lk
g
t
t
= P nj i P l k
(7)
k
f
tj
tk

69

Q
Q
Q
Sea D(t) = (t i ) (t j ) (t k ), tenemos que deg D = n0 (f gh) y
 D(t) 
deg ti = n0 (f gh) 1, similarmente para j y k .
Si multiplicamos la ecuacin (7) por

D(t)
D(t)

obtenemos:

P m
P lk 
i

g
Polinomio de grado n0 (f gh) 1
t
t D(t)
=
= P i P k 
nj
f
Otro polinomio de grado n0 (f gh) 1
lk
tj
tk D(t)
Como g y f son polinomios primos relativos, necesariamente deg g, deg f
n0 (f gh) 1 y ya que h = f + g deg h n0 (f gh) 1. Finalmente hemos
probado que
max {deg f , deg g, deg h} n0 (f gh) 1

Existen muchas pruebas del teorema ABC para polinomios (teorema


24), la mayor parte de ellas utiliza la nocin de derivada.

8.3.

ABC para los enteros?

Primeramente tendramos que


los anlogos a deg f y n0 (f ) paQr buscar
mi
ra los enteros.
Sea
m

N,
m
=
p
.
Una
posible eleccin podra ser
i=1 i
Pr
deg m = i=1 mi y n0 (m) = r. Sin embargo en retrospectiva sera mejor
corresponder
r
X
deg P log |m| =
mi log pi
|{z} i=1 |{z}
peso

altura

n0 (P ) n0 (m) =

r
X

log pi

i=1

o bien N0 (m) =

Y
p|m

p=

r
Y

pi = en0 (m) (el radical de m).

i=1

La altura puede ser grande an si n0 es pequeo, por ejemplo si m = 21000


tenemos log 21000 = 1000 log 2 > n0 (21000 ) = log 2.
Conjetura ABC (Masser-Oesterl 1986)

70

> 0, C tal que a, b, c enteros relativamente primos con a + b = c se


tiene
max {log |a|, log |b|, log |c|} (1 + )n0 (abc) + C
o de manera equivalente
0 (abc)1+
max {|a|, |b|, |c|} CN
En agosto del 2012 el matemtico japons Shinichi Mochizuki public un
articulo en su pgina personal afirmando haber demostrado este enunciado. Su prueba de ms de 500 pginas est pendiente pendiente de verificacin por otros matemticos.
Observemos que es no es posible tener = 0 en general. En efecto, si ton
n
mamos an = 32 , bn = 1 y cn = 32 1, tenemos que an + bn = cn con an , bn
y cn relativamente primos. Adems si n > 0, 2n+2 | cn , luego
N0 (an bn cn ) 3 2

|cn |
3|an |
n+1
n+2
2
2

en consecuencia tenemos
max {|an |, |bn |, |cn |} = |an | > 2n+1

N0 (an bn cn )
> 2n1 N0 (an bn cn )
3

luego el cociente
max {|an |, |bn |, |cn |}
> 2n1
N0 (an bn cn )
puede tomar valores arbitrariamente grandes.
Si la conjetura ABC para los enteros es cierta podemos, entre otras cosas,
deducir el ltimo teorema de Fermat en su forma asinttica.
Corolario 9. Para n suficientemente grande, no existe ninguna solucin entera
no trivial de la ecuacin xn + y n = zn
Demostracin. Podemos suponer (caso contrario dividimos por lo que sea
necesario) que si hay una solucin (x, y, z), esta es tal que x, y y z son primos relativos. Si por el absurdo tal solucin existe, es decir xn + y n =
|{z} |{z}
a

z , ABC implica que


|{z}
c

n log |z| (1 + ) log |x||y||z| + K 3(1 + ) log |z| + K


71

K
+ 3(1 + ) la desigualdad de arriba ya no es verdadera y
log |z|
entonces se contradice ABC.
luego si n >

El inters de la conjetura ABC no slo se concentra en la aplicacin


que acabamos de ver. La conjetura ABC implica muchos otros resultados
interesantes en aproximacin diofantina, etc. Sera interesante poder definir una nocin de derivada sobre Z anloga a la de los polinomios para
poder traducir la prueba del teorema 24 para los enteros.
Finalicemos este curso enunciando una conjetura debida a Oesterl de la
cual el teorema ABC para los polinomio es un caso particular:
Conjetura (Oesterl)
Sea (u, v) C[t][u, v] con deg 3 y todas sus races distintas. Existe
C tal que para todos f , g C[t] relativamente primos
(d 2) deg((f , g)) n0 ((f , g)) + C
Esta conjetura implica el teorema 24 (que ya demostramos), simplemente
tomamos = u 2 v + uv 2 = u
v ( u + v ), d = 3 y C = 1 (claro est,
|{z} |{z} |{z}
a

habra que mostrar que en tal caso C de la conjetura es tal que C 1 y


rebautizarla como C = 1).

72

9.
9.1.

Soluciones e indicaciones de las series de ejercicios


Serie de ejercicios 1

1. Observemos que si p > 2 es primo, entonces p es de la forma 4k + 1 o


bien de la forma 4k + 3 segn la divisin euclidiana. Supongamos por el
absurdo que los primos de la forma 4k + 1 son finitos, sea {p1 , ..., pn } la lista
de tales primos. Suponiendo que p1 = 3, sea x = 4p2 ...pn +3. Entonces, pi - x
para todo i = 2, ..., n, sino pi | 3. Como el producto de nmeros de la forma
4k + 1 es tambin de la forma 4k + 1, sigue que x debe ser divisible por
un nmero primo p de la forma 4k + 3. Como x no es divisible por 3, p no
pertenece a la lista {p1 , ..., pn } y por lo tanto es un nuevo nmero primo de
la forma 4k + 3 ms grande que los precedentes.


2. Utilizar la identidad: a2k+1 + 1 = (a + 1) a2k a2k1 + a2k2 + 1 .
3. (i) Escribamos m = n + k con k 1. Tenemos que


 m
  n+k   n+(k1)
2
2 1
2
2n+(k1)
2 1 = 2
= 2
1 2
1 .
n+(k1)

Repitiendo esta factorizacin cuantas veces


sea necesario
sobre 22
1


n
m
n
m
2
2
2
2
deducimos que 2 + 1 | 2 1. Sea d = 2 + 1, 2 + 1 et mximo comn
m

divisor de ambos nmeros. Como d | 22 + 1 y tambin d | 22 1, entonces


m
n
d | 2. Luego, d = 1 ya que 22 + 1 y 22 + 1 son impares.
(ii) De la demostracin del teorema 2, tenemos que pn+1 p1 pn + 1.
Utilizamos esa desigualdad y la relacin
n1 

Y
 n

2
2k
x 1 = (x 1)
x +1
k=0
n

para mostrar por induccin que pn+1 22 + 1. Luego, observemos que


(x)
x < p(x)+1 . Sigue entonces que x 22 ya que en realidad se puede man
yorar pn+1 con 22 , de esta manera, aplicando logaritmos se tiene el resultado pedido.
4. El mnimos comn mltiplo de dos nmeros a y b es el nmero ms

pequeo que es divisible por a y b. Si a = p1 1 pk k , b = p11 pk k , entonces


max(1 ,1 )

mcm(a, b) = p1

73

max(k ,k )

pk

Utilizar luego la observacin 2.


5. Mostrar por induccin que si n 2, entonces Hn =
bn par no nulos.

9.2.

an
bn

con an impar y

Serie de ejercicios 2

1. Sean {p1 , ..., pk } los factores primos distintos de n. La funcin solo contabiliza la contribucin de aquellos divisores de n donde no se repita un
pi . As, el divisor pi1 pil , contribuye con |(pi1 pil )| = |(1)l | = 1 y hay

en total kl de ellos en n. Luego
!
!
!
X
k
k
k
|(d)| = 1 +
+
+ +
= 2k .
1
2
k
d|n

2. Las funcionesP y son multiplicativas, luego es multiplicativa y por


lo tanto g(n) := d|n (d) (d) es multiplicativa por el lema 3. Entonces, slo
necesitamos conocer el valor de g en una potencia de un primo pj :
 
g pj = (1) (1) + (p) (p) = 1 (1 + p) = p.

Finalmente, si n = p1 1 pk k , se tiene
g(n) = (1)k p1 pk .
X
3. Por el lema 5, tenemos Id(n) =
(n), luego por la formula de inverd|n

sin de Mbius (n) =

X
d|n

 
(n) X (d)
n
=
.

(d)Id
d
n
d
d|n

Si Re s > 1, tenemos



X (n) X
X ( 1)(n)
1(n)

ns
ns
ns
n=1
n=1
n=1
| {z }

lema 4

X
(n)
n=1

= 1,

(s)

lo que nos da la relacin

X
(n)
n=1

ns

74

1
.
(s)

(8)

Necesitaremos tambin la siguiente estimacin


X1
= log x + O(1).
d

(9)

dx

Para una justificacin


de la frmula 9, comparar la suma de la izquierda
Rx
du
con la integral 1 u .
Ahora ya disponemos de todos los ingredientes para este ejercicio:
X (n) X X (d) X X (d) X (d) X
=
=
=
1
n
d
d
d
x
x
nx
nx
dx m d

d|n

dx

m d

|{z}
[x/d]

X (d)  x
dx


X (d)
X (d)
x
{ } = x
+
O(1)
.
d
d
d
d2

dx

dx

Para la primera suma tenemos


X (d)
dx

d2

X
(d)
d=1

d2

 
 
 
1
1 (8) 1
1
6
+O
+O
=
= 2 +O
.
x
(2)
x
x

Y para la segunda


X (d)
X (d) X 1 (9)


= log x + O(1)
= O(log x).
d
dx d dx d
dx
Finalmente

X (n)
n

nx

6
x + O(log x).
2

4. i) Sea n = p1 1 pk k la descomposicin de n es factores primos distintos.


Solamente los divisores de n de la forma p contribuyen a la suma con

log p. Por cada pi que divide n, los divisores pi , pi2 , ..., pi i contribuyen con
i log p, luego
X

(d) =

k
X




i log pi = log p1 1 pk k = log n.

i=1

d|n

En particular, hemos mostrado que 1 = log.


ii) Observemos que

(n)
n1 ns

P P

(s) =

log p
m1 pms .

Por el teorema 4, tenemos

(1 ps )1

75

si Re s > 1. Luego log (s) =


ambos miembros, tenemos

s 1 =
p log (1 p )

P P
p

pms
m1 m .

Derivando

0 (s) X X log p X (n)


=
=
.
ms
s
(s)
p
n
p
m1

9.3.

n1

Serie de ejercicios 3

1. PodemosPsuponer |x| < 1 y observar que la derivacin trmino a trmino


n
de la serie
n=1 x nos da
X
x
.
(10)
nxn =
(1 x)2
n1

La frmula (10) puede ser considerada como una igualdad formal de polinomios, luego no es necesario poner una condicin sobre x. Tenemos entonces

X
X
X
X
X
n

n k

(x ) =
(n)
=
(n)
(d) xm

1x
n=1
n=1
m1 d|m
k=1
| {z }
=

X
m1

x
.
mx =
(1 x)2

2. Multiplicando cada ecuacin por las inversas de 2,3 y 5 mdulo 3,5 y 7


respectivamente, transformamos el sistema de congruencias en el sistema
equivalente
x 2 (mod 3)
x 2 (mod 5)
x 3 (mod 7)
Para poder aplicar el teorema chino del resto (teorema 5) debemos calcular
los valores: M1 = 5 7 y1 = 2, M2 = 3 7 y1 = 1 y M3 = 3 5 y1 = 1.
Luego la solucin es
x = y1 M1 c1 + y2 M2 c2 + y3 M3 c3 = 140 + 42 + 45 = 122 17 (mod 105).
3. Ya que k 1, akpk+1 a (mod p) es una consecuencia directa del pequeo teorema de Fermat (teorema 7).

76

Sea 798 = 2 3 7 19 la descomposicin en factores primos de 798. Observemos que para cada divisor primo p de 798, existe un entero k 1 tal que
kp k + 1 = 19. Por ejemplo, para p = 7 tenemos k = 3. Luego, por lo visto
anteriormente p | akpk+1 a = a19 a. Deducimos entonces que 798 | a19 a.
4. Buscamos enteros a, b, c, d y e tales que para todo entero x, al menos
una de las congruencias:
x a (2)
x b (3)
x c (4)
x d (6)
x e (12)

(A)
(B)
(C)
(D)
(E)

sea satisfecha. Como nuestra eleccin es arbitraria, podemos empezar poniendo a = 0 (esto supone quitar todos los x impares). Luego podemos
poner b = 0, enseguida podemos poner c = 1 (o bien c = 3, pero no 2 ni
4 porque x satisfacera la ecuacin (A) y lo que queremos es cubrir todas
los posibles valores de x para que se verifique al menos una de las congruencias). Si x = 0, 2, 4 (6), x satisface (A) y si x = 3 (6), x satisface (B). En
consecuencia, es mejor poner d = 1 o d = 5.
Finalmente, para hacer la mejor eleccin de e, elegimos un valor que no
satisfaga alguna de las otras congruencias. As, si e = 0, 2, 4, 6, 8, 10 (12) entonces x verifica (A), si e = 3, 9 (12) entonces x verifica (B), si e = 1, 5 (12)
entonces x verifica (C) y si e = 7 (12) entonces x verifica (D). Luego, lo mejor es poner e = 11. Vemos entonces que hemos logrado cubrir todos los
casos.

9.4.

Serie de ejercicios 4

1. Ver el ejercicio 2 de la serie 3.


2. Podemos suponer sin perdida de generalidad que a1 = 0 y b1 = 0. Supongamos por el absurdo que {a2 b2 , ..., ap bp } {0} es un sistema completo
de residuos mdulo p, por el teorema de Wilson (proposicin 8) tenemos
que
p
Y
ai bi 1 (mod p).
i=2

77

Qp
Qp
Qp
De la misma forma, i=2 ai = i=2 bi 1(mod p). Luego i=2 ai bi 1(mod p),
lo cual no es posible.
p1

3. Observemos que 1, 2, ... 2 , 1, 2, ...,


p no nulos, luego por la proposicin 8:
(1)

p1
2

p1
2

son todos los residuos mdulo

x2 = (p 1)! 1 (mod p),

de donde
x2 (1)

p+1
2

(mod p).

4. Observemos que la indicatriz de Euler posee la propiedad siguiente


Sea n > 0 un entero, (n) = n 1 n es primo.

(11)

Por hiptesis a , 0. Como n > 2 y a an2 1 (mod n), sigue que a (Z/nZ) .
Luego, por la proposicin 6, a(n) 1 (mod n). Sea r el orden de a, por
definicin de orden r | n 1. Por hiptesis r = n 1 y r = n 1 | (n). Ya que
(n) n 1, necesariamente (n) = n 1 y luego, por (11), n es primo.

9.5.

Serie de ejercicios 5

1. Observemos que 3 es un elemento primitivo en (Z/7Z) ya que 31 = 3,


32 = 2, 33 = 6, 34 = 4, 35 = 5 y 36 = 1. Tomando logaritmos discretos en
esta base:
2 log y log 5 + 3 log x (mod 6)
2 log y 5 + 3 log x (mod 6).
Poniendo X = log x y Y = log y obtenemos
(X, Y ) {(1, 1), (1, 4), (3, 1), (3, 4), (5, 1), (5, 4)}.
De donde derivamos las soluciones de la ecuacin original:
(x, y) {(3, 3), (3, 4), (6, 3), (6, 4), (5, 3), (5, 4)}.

2. Sea g un elemento primitivo de (Z/pZ) . Tenemos que


p
X
n=1

p1
X

(g k )l (mod p).

l=1

78

Luego si p 1 | k, entonces g k 1 (mod p) segn la proposicin 7. Por lo


tanto

p1

X
1 (mod p) si p 1 | k

p kp1
k
(g k )l
.
(g
)
g

0 (mod p)
sino

l=1

g k 1

3. Con las mismas notaciones nde la seccin


la lista
i h
i de los represeno
h p14.3,
p1
tantes ms pequeos de j 2 es 2, 4, 6, ..., 2 4 , 2 4 + 1 p, ..., 3, 1 ,
h p1 i
p1
luego l = 2 4 . Entonces, l es par si p 1 (mod 8) y en ese caso
 
 
2
2
=
1,
l
es
impar
si
p

3(mod
8)
y
en
tal
caso
p
p = 1. Lo que podemos
resumir escribiendo:
!
p2 1
2
= (1) 8 .
p
4. Aplicando las propiedades del smbolo de Legendre y de la reciprocidad
cuadrtica:
      
 
731 51 73
31 731
15
3
5
73
2
2
=
=
(1)
(1) 2 2
73
73 73
5
3
      
32 1
5
2
3 1
=
=
= (1) 8 = 1.
=
5 3
3
3
0
 p = 4k + 3, entonces p = 2p + 1 = 8k + 7. Luego por el corolario 3,
5. Si
2
0
2
0
p0 = 1. Existe entonces a no divisible por p tal que 2 a (mod p ). Luego
por el pequeo teorema de Fermat:
0

2p a2p ap 1 1 (mod p0 ).
Como p = 251 = 6 4 + 3 es primo y p0 = 2p + 1 = 503 tambin lo es. Luego
p0 = 503 | 2251 1. Por lo tanto 2251 1 no es primo.

9.6.

Serie de ejercicios 6

1. Supongamos que p , 2, 5 (en ambos casos 5 es un residuo cuadrtico).


Por la ley de reciprocidad cuadrtica
! 
51 p1
5 p
= (1) 2 2 = 1.
p 5
Luego 5 es un residuo cuadrtico mdulo p si y solamente si p es un residuo cuadrtico mdulo 5. Por el criterio de Euler (proposicin 11):
 
51
p
p 2 = p2 (mod 5).
5
79

Tenemos que determinar entonces los primos p tales que p2 1 (mod 5).
Calculando mdulo 10 deducimos que
! (
5
1 si p 1, 9 (mod 10)
=
1 si p 3, 7 (mod 10)
p
2. Aplicando las propriedades se obtiene

123
917

= 1.

3. Si p = 2n + 1 es primo, entonces g es primitivo si y solamente si

g 
p

= 1.

) Supongamos por el absurdo que q x2 (mod p). Como g es primitivo,


existe un entero l tal que x g l (mod p). Luego se tendra g 2l1 1 (mod p)
y entonces ord(g) = p 1 = 2n | 2l 1 lo cual no es posible.
) La contrapuesta es una reformulacin de la proposicin 11.
Supongamos que n > 1. Entonces p = 2n + 1 > 3 es un primo impar. Por
otra parte, la ley de reciprocidad cuadrtica nos dice que
! 
(2n +1)1
31
3 p
= (1) 2 (1) 2 = 1,
p 3
es decir 3 es un residuo cuadrtico mdulo p si y solamente si p = 2n + 1 es
un residuo cuadrtico mdulo 3. Para deducir que 3 es un elemento primitivo mdulo p, ser suficiente mostrar que p no puede ser un residuo cuadrtico mdulo p. Supongamos por el absurdo que p = 2n + 1 x2 (mod 3).
Observemos que 3 - x y que todos los cuadrados no divisibles por 3 son
congruentes a 1 mdulo 3. Por otra parte, para todo n 0, 2n 1, 2(mod 3),
lo que implica que 2n + 1 2, 0 (mod 3). Deducimos entonces que p no es
un cuadrado mdulo 3 y por lo visto anteriormente 3 es un elemento primitivo mdulo p.
4. Como a . 0 (mod p) y p > 2, a y 2 son invertibles mdulo p. Luego la


b 2
4ad 2 (mod p).
ecuacin ax2 + bx + c 0 (mod p) es equivalente a x + 2a
Por el teorema de lagrange esta ecuacin tiene a lo sumo 2 soluciones y
de hecho posee exactamente dos soluciones si y slo si d es un cuadrado
mdulo p.
5. Sea la afirmacin A : Si n 6, entonces n = p +q +r con p, q y r primos.
Denotemos por G la conjetura de Goldbach G : Todo numero par mayor o
igual a 4 se escribe como suma de dos primos. Entonces A G. En efecto,
G A) Si n 6 es impar, entonces n 3 4 es par y por G, n = 3 + p + q con
80

p y q primos. Si n 6 es par, entonces n 2 4 es par y por G, n = 2 + p + q


con p y q primos.
A G) Si n 4 es par, entonces por A, n + 2 = p + q + r con p, q y r primos.
Como n + 2 es par, p, q y r no pueden ser simultneamente pares, luego sea
q = 2, sea p = 2, sea r = 2. Finalmente, podemos suponer sin prdida de
generalidad que n = p + q, lo que muestra G.

9.7.

Serie de ejercicios 7

1. g(x, y) = x2 !+ 4xy + 5y 2 = (x + 2y)2 + y 2 = f (x + 2y, y), es decir g = f


1 2
con =
SL2 (Z). Luego los enteros de la forma x2 + 4xy + 5y 2 son
0 1
todos los enteros positivos que se escriben como suma de dos cuadrados,
los cuales estn caracterizados en el teorema 9.
2. Sea n = 5 y definamos las formas cuadrticas siguientes
f (x1 , ..., x5 ) =

5
X

xi2 , g(x1 , ..., x5 ) =

i=1

4
X

xi2 + 2x52 .

i=1

det Mf = 1, det Mg = 2 f  g. Las imgenes de f y g son enteros positivos y el teorema 10 de la suma de cuatro cuadrados (poniendo x5 = 0)
nos dice que son exactamente todos los enteros positivos, es decir f (Z2 ) =
g(Z2 ) = Z0 .
3. Los cuadrados son congruentes a 0, 1, 4 mdulo 8, luego la suma de tres
cuadrados es congruente a 0, 1, 2, 3, 4, 5, 6 mdulo 8. Deducimos entonces
que {8l + 7 | l Z0 } E. Por lo tanto
1
1
lm sup |{a x | a E}| lm sup |{0 x | 7(8)}|
x x
x x


1 x+7
lm sup
8
x x



1 x+7
x+7
lm sup

8
8
x x
1
=
8
4. Supongamos por el absurdo que 22k+1 = a2 + b2 + c2 + d 2 con a b c
d > 0 enteros. Como 22k+1 0 (4), a, b, c, d 0 (2) o bien a, b, c, d 1 (2).
El segundo caso, es decir cuando a, b, c y d son impares no es posible ya
81

que a2 + b2 + c2 + d 2 4 (8), lo que contradice el hecho que 22k+1 0 (8)


(k 1). Finalmente el primer caso se reduce al segundo dividiendo por 4
la igualdad 22k+1 = a2 + b2 + c2 + d 2 tantas veces como sea necesario para
que a, b, c y d sean todos impares, es decir hasta que k sea igual a 1.

9.8.

Serie de ejercicios 8

1. Sea = 3,1415926. Aplicamos el algoritmo comenzando con 0 = y


a0 = [0 ] = 3:
1
= 7,06251597894240 a1 = [1 ] = 7,
0 a0
1
2 =
= 15,9959104362960 a2 = [2 ] = 15,
1 a1
1
3 =
= 1,00410635691218 a2 = [3 ] = 1.
2 a2
1 =

Luego los primeros cuatro convergentes de son


p
p0
22 p2 333 p3 355
= 3, 1 =
,
=
,
=
.
q0
q1
7 q2 106 q3 113
2. Sea = [a0 , ..., an ] conh ai0 0. Tenemos que 1 < 1 ya que ai 1 para todo
1
i = 0, ..., n. Luego b0 = 1 = 0 y seguidamente 1 = 1/b
= , de donde
0
b1 = [1 ] = a0 y por lo tanto 1 = [b0 , b1 , ..., bn+1 ] = [0, a0 , a1 , ..., an ].

3. Como n < n2 + 1 < n + 1, entonces a0 = [ n2 + 1] = n. Luego, aplicando


el algoritmo:

1
1
1 =
=
= n2 + 1 + n a1 = [1 ] = 2n,
a0
n2 + 1 n

1
1
2 =
=
= n2 + 1 + n a2 = [2 ] = 2n,
1 a1 ( n2 + 1 + n) 2n
vemos que la sucesin ai se vuelve constante igual a 2n a partir de i 1.
Por lo tanto n2 + 1 = [n, 2n].
4. Sea p(x) = x2 ax 1 con a N . La ecuacin p(x) = 0 posee dos races reales diferentes, una estrictamente positiva 1 > 0 y la otra 2 < 0
estrictamente negativa. Para la raz positiva se tiene
12 a1 1 = 0 1 = a +

82

1
= a+
1

1
1
a+
a +

Refirindonos a la demostracin del teorema 13,


Luego 1 = [a, a, ...] = [a].
tenemos la relacin qn = aqn1 + qn2 con q0 = 1 y q1 = a (observar que
p1
a2 +1
2
q1 = a , de donde q1 = a ya que (a + 1) a(a) = 1 y entonces por la proposicin 2, a2 + 1 y a son primos relativos). Hay esencialmente 2 maneras
de resolver la ecuacin de recurrencia qn = aqn1 + qn2 .
Primera forma: Considerar la relacin matricial
!
!
!
qn
a 1 qn1
=
qn1
1 0 qn2
que iterndola nos da
!
!n1
!
qn
a 1
q1
=
,
qn1
1 0
q0
luego diagonalizamos la matriz en cuestin para evaluar su (n 1)-sima
potencia y as poder calcular qn .
Segunda forma: Buscar una solucin de la forma qn = Ar n con A una constante que determinaremos con las condiciones iniciales q0 = 1, q1 = a. Remplazando qn por Ar n en la ecuacin de recurrencia tenemos r 2 r 1 = 0.
Esta ecuacin tiene como races 1 y 2 . La homogeneidad de la ecuacin
de recurrencia implica que qn = A1n + B2n es la solucin general. Con las
2
1
, B = 2
. Finalmente
condiciones iniciales determinamos A = 2
1
1
1n+1 2n+1
qn =
.
1 2
La sucesin de Fibonacci est definida por F0 = 0, F1 = 1, Fn = Fn1 + Fn2
para n 2. Luego poniendo qn1 = Fn , a = 1 obtenemos
1
Fn = ( n n ),
5
donde es la raz positiva de x2 x 1 = 0 (conocido popularmente como
el nmero de oro) y = 1 .

9.9.

Serie de ejercicios 9

1. Sea = [1, 2, 3, 4]. Pongamos = [3, 4]. Tenemos que


= 1+

1
3 +
=
.
2 + 2 +
83


1
2 = 12, de donde = 12.
Luego = 3 + 4+1/

2. Sea = [a0 , a1 , ...] irracional y = 1+2 5 . Tenemos que mostrar que qn


n1 . Procedemos por induccin sobre n:
i. n = 0, n = 1 se verifican trivialmente.
ii. n n + 1 : satisface 2 1 = 0, luego
qn+1 = an+1 qn + qn1 n1 + n2 = n2 ( + 1) = n .
| {z }
2

Supongamos ahora que an A para todo n. El nmero = 12 (A + A2 + A)


satisface 2 A 1 = 0. Luego un razonamiento por induccin como antes
permite de concluir que qn n .
3. Sea = [a0 , a1 , ...] con ai ai+1 para todo i = 0, 1, 2, .... Mostremos primero por induccin que qn (an + 1)n para todo n. Los casos n = 0, n = 1
se verifican trivialmente. Supongamos que qk (ak + 1)k para todo k n,
entonces
qn+1 = an+1 qn + qn1 an+1 an+1 ( an +1)n + ( an1 +1)n1
|{z}
|{z}
an+1
n

(an+1 + 1)

an+1

(an+1 (an+1 + 1) + 1)
|
{z
}
(an+1 +1)2

(an+1 + 1)n+1 .
2

Si adems an+1 (an + 1)n para todo n, entonces es trascendente. En


efecto, supongamos por el contrario que sea algebraico de grado d. Por
el teorema de Liouville, existe una constante c = c() > 0 tal que


pn c .

qn qnd
Por otra parte, qn+1 = an+1 qn + qn1 > an+1 qn y luego por el teorema 13, se
tiene tambin


pn 1 < 1 1 .

qn qn qn+1 an+1 qn2 qn2+n
84

La ltima desigualdad es una consecuencia de la hiptesis adicional: an+1


2
(an + 1)n qnn . En resumen tenemos


pn
c
1

2+n ,
d
qn
qn
qn
d(2+n)

para todo n, lo cual es posible solamente si c = 0.


de donde c qn
Esto contradice el teorema de Liouville, por lo tanto es trascendente.
2

4. Sea = ab + ab + ab + . El criterio del cociente nos dice que R.


Sean
2

m = ab + ab + + ab
qm = ab

m1

m1

Q,

N, pm = qm m N.

Sea  = b con > 0 tal que u := 2b + < 1. Esto es posible ya que b 3.


Luego, para todo c > 0 podemos encontrar m tal que
m

ab + ab

m+1

+ < caub .

m+1

En efecto, para todo m 1, ab + ab + converge hacia un nmero


real acotado por . Como u < 1, para todo c > 0 se puede elegir m 1 tal
m
que < caub . Finalmente, existe una infinidad de m tales que


pm
c
c
c


2+b +2 .
| m | =
m1
(2+b)b
qm
qm
a
qm
Luego por el corolario 4, es irracional. Adems no puede ser algebraico, sino se contradicera el teorema de Roth. Por lo tanto es trascendente.

9.10.

Serie de ejercicios 10

1. Como 2 2 (mod 4), se tiene OQ(2) = Z[ 2]. Para que x + y 2 Z[ 2]

sea una unidad, es suficiente y necesario que N (x + y 2) = x2 2y 2 = 1


tenga solucin en los enteros. Luego de unos pocos ensayos, obtenemos

una solucin (x, y) = (3, 2). Finalmente, los enteros algebraicos (3 + 2 2)n ,
n 0 nos proporcionan una
infinidad de unidades diferentes.
nDe manera

anloga, para OQ( 3) = Z[ 3] los enteros algebraicos (2 + 3) , n 0 nos


dan una solucin.

2. Sea K = Q( 6). Como 6 2(mod 4), OK = Z[ 6]. Mostremos que 2 es


85

irreducible en este anillo: Si 2 = 4 = N ()N () de donde N () = 2


o N () = 4. En el segundo caso N () = 1 y entonces es una unidad.
El el primer caso, la ecuacin x2 + 6y 2 = 2 no tiene solucin en los enteros.
Sigue entonces que 2 es irreducible. De forma similar se prueba que
3 y 6 son irreducibles.
Queda por ver que no estn asociados entre si.

Por ejemplo
2 y 6 no estn asociados, sino existira una unidad tal
que 2 = 6, luego pasando a normas 4 = N ()6 y entonces no sera
una unidad. Finalmente, hemos probado que la descomposicin en
factores irreducibles no es nica (mdulo una unidad) y por lo tanto Z[ 6] no
es factorial.
3. Por ejemplo, 2 no es irreducible en OQ(3) . De hecho todos los facto
res de izquierda
y derecha no son irreducibles. En efecto, sean = 1 + 3

y = 4 + 3 3, entonces

= 2, = 11, = 5 + 3, = 5 3.

= ()( )
= (5 + 3)(5 3).

Luego 2 11 = ( )(
)

4. Sea K = Q( 10), entonces OK = Z[ 10]. Observemos que para todo


entero a se tiene que a2 0, 1, 4 (mod 5) y por lo tanto no existen enteros algebraicos en OK con norma 2, 3 ya que una solucin entera de
x2 10y 2 = 2, 3 implicara
una solucin mdulo 5. Por otra parte el en
en
tero algebraico 4 + 10 es irreducible ya que no hay enteros algebraicos

OK con norma 2. Este mismo razonamiento muestra que 4 10 tambin


es irreducible. Consideremos ahora el producto

(4 + 10)(4 10) = 6 = 2 3.
Por los motivos anteriormente mencionados 2 y 3 son irreducibles, luego
OK no es factorial.

9.11.

Serie de ejercicios 11

1. Con las mismas notacionesdel teorema 20


pero
esta
vez
con
K
=
Q(
3)

tenemos = + , = (r + 3s) OK = Z[ 3]. Luego es suficiente observar que r 2 3s2 max{r 2 , 3s2 } 34 < 1.

2. Sean A = 5 + 4 3, B = 1 + 2 3. Entonces A/B = 19/11 + 6/11 3, luego


elegimos r = 2 y s = 1 queson los enteros ms prximos
a 19/11 y 6/11 res

pectivamente. Sea = 2+ 2. Entonces B = 8+5 3 y := AB = 3 3.


86

Por construccin N () < N (B) y es el resto de la divisin


euclidiana
de A
y B. Continuando la divisin
tenemos
B/
=
1/2

5/6
3. Poniendo

E = 3 y Z = B E = 2 3, tenemos por construccin B = E + Z con


N (Z) < N (). De hecho N (Z) = 1, lo que significa que Z es una unidad y
entonces detenemos las divisiones sucesivas. Recapitulando
Z = B E
= B (A B )E
= (1 + E)B EA.
Por lo tanto
1 = Z 1 (1 + E)B Z 1 EA

1 = (2 + 2 3)B + (3 2 3)A.

3. Primeramente calculamos 39 = [6, 4, 12], de donde el periodo m = 2.


Sabemos por el teorema 22 que todas las soluciones x2 19y 2 = 1 estn
dadas por los convergentes de (pn , qn ) tales que n es impar de la forma
n = ml 1 = 2l 1. Luego la solucin positiva minimal esta dada por
(x, y) = (p1 , q1 ) = (25, 4).

4. Calculamos 31 = [5, 1, 1, 3, 5, 3, 1, 1, 10]. Vemos entonces que el periodo m = 8 es par, por lo tanto la ecuacin x2 31y 2 = 1 no tiene soluciones
segn el teorema 23.

87

También podría gustarte